Content Knowledge- Elem Ed CKT

Ace your homework & exams now with Quizwiz!

Introductory Phase

A group of words that come before the main clause in a sentence to help the reader understand more about the main clause. An introductory phrase is not a complete clause; it does not have a subject and a verb of its own.

Graphophonemic cues

(Visual cues) letter-sound information that readers process during reading. May allow readers to mistake 'moving' and 'movies' if relied on as the words share some letters and a similar construction.

Affix

A grammatical element, in the form of a group of letters that is added to the main part of a word, also, known as the root in order to create modified forms of the word. (A prefix or a suffix)

Mr. Schroeder asked his students to find the product 36×5. Four students shared their strategies with the class, and Mr. Schroeder recorded their methods on the board. Indicate whether each of the following representations of student methods demonstrates that the student used reasoning based on the distributive property or reasoning based on the associative property. Representation of Student Method: 1) 36÷2=18 5×2=10 18×10=180 2) 36=30+6 30×5=150 6×5=30 150+30=180 3) 36=6×6 6×5=30 6×30=180 4) 36=40−4 40×5=200 4×5=20 200−20=180

1) Associative 2) Distributive 3) Associative 4) Distributive The first and third representations demonstrate that the student used reasoning based on the associative property, and the second and fourth representations demonstrate that the student used reasoning based on the distributive property. The first method can be represented as 36×5=(18×2)×5=18×(2×5)=18×10=180, which shows how the student used reasoning based on the associative property. The second method can be represented as 36×5=(30+6)×5=30×5+6×5=150+30=180, which shows how the student used reasoning based on the distributive property. The third method can be represented as 36×5=(6×6)×5=6×(6×5)=6×30=180, which shows how the student used reasoning based on the associative property. The fourth method can be represented as 36×5=(40−4)×5=40×5−4×5=200−20=180, which shows how the student used reasoning based on the distributive property.

Direct Scaffolding

A process through which a teacher adds support for students in order to enhance learning and aid in the mastery of tasks. The teacher does this by systematically building on students' experiences and knowledge as they are learning new skills. (e.g A student omits words when reading, the teacher points to teach word to guide the student to recognize the error.)

R- Controlled Vowel

Any vowel followed by an r. The r changes the sound that vowel makes

Morpheme

in a language, the smallest unit that carries meaning; may be a word or a part of a word (such as a prefix)

What is another name for a two-part test question?

Composite.

Types of Miscues

Correction- A common sign of a competent reader, a correction is a miscue that the student corrects in order to make sense of the word in the sentence. Insertion- An insertion is a word(s) added by the child that is not in the text. Omission- During oral reading, the student omits a word that changes the meaning of the sentence. Repetition- The student repeats a word or portion of the text. Reversal- A child will reverse the order of the print or the word. (from instead of form, etc.) Substitution- Instead of reading the word in the text, a child substitutes a word that may or may not make sense in the passage.

Students are presenting reports on why certain organisms survive better in certain environments. One student reports on the Arctic environment. Of the four underlined terms, click on the one improperly used in the student's report. Polar bears can survive in the cold and ice because of their adaptations. They have thick fur and are white. If any black polar bears are born, they evolve after they're born and become white. The bears are carnivores because there are no plants growing where they live. Their prey include penguins and seals. "Polar bears can survive in the cold and ice because of their adaptations. They have thick fur and are white. If any black polar bears are born, they evolve after they're born and become white. The bears are carnivores because there are no plants growing where they live. Their prey include penguins and seals."

Evolve is the correct selection. An individual bear does not "evolve" from one color to another. The student is misusing the word, and the phenomenon described does not occur.

Match each fraction with its equivalent decimal number. Decimals: 0.02, 0.2, 2 Fractions: A. 20/100 B. 2/10 C. 20/10 D. 200/100 E. 2/100

First column: E; Second column: A, B; Third column: C, D. The correct answers are that 2/100 is equivalent to 0.02, 20/100 and 2/10 are equivalent to 0.2, and 20/10 and 200/100 are equivalent to 2. The decimal equivalent of any fraction can be found by dividing the numerator of the fraction by its denominator. Please note that credit for the correct answer is given regardless of the boxes in which the fractions are placed when matched with their equivalent decimal numbers.

Word Families

Groups of words that have a common pattern or groups of letters with the same sound.

Directionality

How to read from left to right.

Syntactic Cues

Involve word order, rules and patterns of language (grammar), and punctuation. For example, the position a word holds in a sentence will cue the listener or reader as to whether the word is a noun or a verb.

Vowel-consonant-e (VCe)

Syllables contain a silent e at the end that elongates the vowel sound. Mat => Mate

A fourth-grade teacher plans a lesson to analyze the Pilgrims' voyage to the New World from England in search of religious freedom in the seventeenth century. The teacher then incorporates an extension activity based on an article on current events to help students compare the Pilgrims with European immigrants leaving their countries in the 1800s for better economic opportunities. Which of the following is the teacher most likely attempting to accomplish through the lesson? A.Demonstrating how themes from the past recur in modern times B.Revealing that solving problems from the past prevented them from recurring C.Illustrating how people's attitudes toward immigration changed over time D.Focusing on how countries change their policies based on past events

Option (A) is correct. Both the historical lesson and the modern link demonstrate why people move from their homes in search of other, better lands—a theme which recurs over time and in many contexts.

Students completed a simulation examining the erosion of soil by water and made several conclusions. Which of the following student conclusions presents a misconception? A."Erosion is always harmful to the environment." B."Erosion involves the movement of material from one place to another." C."The slope of the land affects erosion." D."The amount of vegetation affects erosion."

Option (A) is correct. Erosion has several environmental benefits, such as the creation of new soil, the formation of new habitats, and the transport of carbon to wetland areas.

After completing a unit on citizenship, a fifth-grade teacher asks students to design a community service project. Which of the following student projects most accurately fulfills the end-of-unit assignment? A.Collecting paper trash from neighbors to take to the local recycling plant B.Helping to wash cars as a fund-raiser for the class field trip C.Contacting the state senator from the district about creating a new park D.Volunteering to do more tasks at home to help the family

Option (A) is correct. Helping neighbors with their trash by taking paper trash to a local recycling plant demonstrates responsible citizenship.

Which of the following word problems can be represented by the equation 4×n+8=16 ? A.A set of 5 baskets holds a total of 16 apples. The first basket has 8 apples and the other baskets each hold an equal number of apples. How many apples are in each of the other baskets? B.There are 12 baskets, 8 of which are empty. There are 16 apples, with an equal number of apples in each of the other 4 baskets. How many apples are in each of the 4 baskets? C.There are 16 baskets, 8 of which are empty. Each of the other baskets contains 4 apples. How many apples are there in all? D.There are 8 baskets with 4 apples in each basket and 16 apples that are not in a basket. How many apples are there in all?

Option (A) is correct. If there are 5 baskets and one basket holds 8 apples, the rest of the apples are split evenly among the other 4 baskets. Therefore, to find the number of apples in each of the 4 baskets, the equation 4×n+8=16 can be set up, where n is the number of apples in each of the 4 baskets. The problem in option (B) can be represented by the equation 4×n+8×0=16, the problem in option (C) can be represented by the equation 4×(16−8)=n, and the problem in option (D) can be represented by the equation 8×4+16=n.

A fifth-grade teacher develops a lesson around the communitarian idea that societies must balance the rights and responsibilities of individuals with the common good. Which of the following instructional tools is the best source to support the objective? A.A video featuring the six essential elements of democracy B.A time line showing important events from 1750 to 1800 C.A map of the thirteen original colonies with the dates when they were formed D.A Venn diagram comparing clothing of indigenous peoples and European settlers

Option (A) is correct. In order to understand that societies must balance the rights and responsibilities of individuals with the common good, students must understand the essential elements of democracy. Showing the students a video of the six elements of democracy will help the students learn the concepts.

Which of the following strategies best uses geography skills to engage students in inquiry-based learning? A.Plotting a national landmark using its latitude and longitude and answering the five "W" questions about the landmark B.Using a globe to identify latitude and longitude lines for various locations C.Completing a worksheet focused on using latitude and longitude lines followed by a short knowledge check D.Taking notes during a teacher-directed lecture on latitude and longitude lines

Option (A) is correct. Inquiry-based learning includes the collection, evaluation, analysis, and interpretation of information. This activity includes all of these components.

Which of the following is an example of modifying the physical environment to adapt to the needs of the people living in a community? A.Planting trees to decrease highway traffic noise B.Introducing digital maps to make navigation easier C.Identifying areas of natural drainage to limit building D.Allowing free access to the local library

Option (A) is correct. It demonstrates how people have modified the physical environment in which they live—planting trees that did not previously exist.

While teaching a unit on light, a teacher asks her students to use pictures and words to explain how they can see the apple on her desk. One of her students draws a picture of the apple with arrows going away from it and toward the eyes drawn on a stick figure. The student explains that the apple glows and sends light to our eyes. What follow-up question would help the student overcome this misconception? A.If you turned off the lights, would you still be able to see the apple? B.How does the color of the apple affect the type of light it creates? C.Why does light travel in a straight line? D.How would your picture change if the apple was behind a window?

Option (A) is correct. It would allow the student to realize that the light must be reflected off the apple.

A fifth-grade teacher is beginning a unit on the Industrial Revolution. Which of the following was an effect of the Industrial Revolution on the United States? A.An increase in urbanization and immigration B.A decrease in the construction of railroads C.An increase in the population of rural areas D.A decrease in the number of women in the workforce

Option (A) is correct. New and efficient manufacturing techniques created opportunities for low-skilled workers to come to rapidly urbanizing areas.

A first-grade teacher introduces a unit on family. Which of the following activities is the most appropriate way to introduce the concept? A.Gathering the students in a circle and reading a book about different types of families around the world B.Brainstorming with the students about how their families can participate in the different school events C.Assigning the students homework where they have to discuss their own family ancestry with a family member D.Having the students compare and contrast different kinds of families using a graphic organizer

Option (A) is correct. Reading a book to introduce the concept is the most appropriate way to introduce a lesson in first grade.

A rectangular message board in Aleyah's dormitory room has a length of 30 inches and a perimeter of 108 inches. A rectangular bulletin board in the hallway outside Aleyah's room is twice as long and twice as wide as the message board. Which of the following statements about the bulletin board is true? A.The bulletin board has a width of 48 inches. B.The bulletin board has a length of 96 inches. C.The area of the bulletin board is twice the area of the message board. D.The perimeter of the bulletin board is four times the perimeter of the message board.

Option (A) is correct. Since the message board has a length of 30 inches and a perimeter of 108 inches, the width of the message board can be found by solving the equation 2(30)+2w=1082(30)+2w=108 for w. To solve the equation for w, subtract 60 from both sides of the equation and then divide both sides of the equation by 2 to find that w=24w=24. This means that the length and width of the bulletin board are 60 inches and 48 inches, respectively, and it can be concluded that the area of the message board is 720 square inches, the perimeter of the bulletin board is 216 inches, and the area of the bulletin board is 2,880 square inches. Thus, the only true statement is that the bulletin board has a width of 48 inches.

Before European exploration in North America, which of the following weapons was used by indigenous people to hunt small game? A.Stone spearheads B.Firearms C.Metal swords D.Steel traps

Option (A) is correct. Spears with barbed points were weapons the indigenous people developed with natural resources such as stones.

Which of the following activities is best for building on students' existing knowledge when starting a new unit on culture for second-grade students? A.Creating a list of traditions practiced in their families B.Taking notes while reading an expository text about different customs C.Comparing different religions practiced around the world D.Describing the development of different languages

Option (A) is correct. Students develop an understanding of traditions and customs by first observing them within their own families and communities.

A third-grade teacher starts a lesson on the first methods used to obtain goods and services to meet individual needs. Which of the following is the most appropriate classroom activity related to the topic? A.Trading goods with classmates without the use of currency B.Reading an informational article about world currencies independently C.Using an online currency converter to learn about exchange rates D.Estimating the cost of household goods during a specific time period

Option (A) is correct. The barter system was used to exchange goods and services prior to the introduction of currencies. Through this method humans have been able to acquire goods to help meet their needs. The activity is also appropriate for third graders to comprehend.

A teacher asks students, "What's cowboy without the cow?" The teacher is promoting which of the following phonological awareness skills? A.Segmenting B.Deletion C.Blending D.Substitution

Option (B) is correct. In the scenario, the teacher is asking students to "delete," or remove, the word part "cow" from "cowboy." This will allow the students to hear and understand the remaining word part "boy," leading to their eventual understanding of the compound word parts.

Which of the following describes a student-created model that best represents the movement of matter through an ecosystem? A.An interconnected web showing the movement of matter between plants, animals, decomposers, and the environment B.A pyramid model showing the relative biomass at each trophic level C.A linear model showing the movement of matter from plants to animals to decomposers to the environment D.A concept map diagramming how a plant uses water, soil, and decomposed materials to grow and reproduce

Option (A) is correct. The movement of matter through an ecosystem is best represented by a model that shows connections between all the organisms in the ecosystem and between the organisms and their environment.

Ms. Garrett has been working on verbal counting with her students. She wants them to be more aware of patterns in the way number names are typically constructed. Which of the following number names LEAST reflects the typical pattern in the way number names are constructed in the base ten system? A.Eleven B.Sixteen C.Twenty-five D.Ninety

Option (A) is correct. The number name "eleven" does not follow any pattern of number-name construction with reference to the tens and ones. Option (C) is not correct because "twenty-five" follows the most typical structure of how number names are constructed for whole numbers, since the number of tens in the number is referred to first, followed by the number of ones. Although the numbers in options (B) and (D) do not follow the most typical structure like "twenty-five" does, where the tens are called out specifically, the numbers in these options do follow a structure of the number of ones being named, followed by "teen," which refers to the ten in the number. Therefore, these numbers follow a pattern, unlike "eleven."

When writing a story, a student spells the words "phone," "boat," and "click" as "fon," "bot," and "clik." Which of the following hypotheses about the student is supported by this pattern of errors? Select all that apply. A.The student believes that each sound is represented by only one letter. B.The student has confused some letter names with the sounds they represent. C.The student thinks that the position of a letter within a word does not affect the sound it makes.

Option (A) is correct. The student has identified the correct sounds and has written them in the correct order, but in each of the words the student makes an error in using only one letter where a blend or digraph is required.

A teacher is holding a classroom discussion after a lesson on the ways in which cities develop to meet economic needs. The teacher asks students to brainstorm economic factors that contributed to an increase in the population of cities like New York and Boston during the late 1800s. Which of the following student responses indicates that the student may require additional instruction? A."People started to move to cities because they wanted to own their own land." B."People started to move to cities to find work in the factories that were being built." C."People started to move to cities to more easily sell goods in international markets." D."People started to move to cities because they couldn't find work in rural areas."

Option (A) is correct. The teacher should focus instruction on the idea that most people did not move to cities with the intention of acquiring land.

After teaching a civics unit, a third-grade teacher divides students into groups, gives each group a packet of cards representing governmental activities and the three branches of government, and asks students to match each governmental activity to the appropriate branch of government. Which of the following is the function of this activity? A.Reinforcing the concept of separate roles and responsibilities within the government B.Examining how the various government offices and agencies collaborate C.Identifying how bills and laws are passed under the legislative department D.Extending knowledge about the constitutional rights and amendments

Option (A) is correct. This activity reinforces the separate roles and responsibilities under the three branches of the government: legislative, executive, and judiciary.

Students are studying various aspects of magnetism. Which of the following questions could upper-elementary students be able to answer experimentally with classroom resources? A.What would cause a compass to not point to the north? B.Why do all magnets have a north and south pole? C.Is navigation using a compass more accurate than using the Sun and stars? D.Why are magnets made of particular materials?

Option (A) is correct. This question can be investigated empirically inside the classroom with compasses, magnets and other objects.

Mr. Shope has his students mix one cup of warm water, one tablespoon of borax powder, and one-fourth cup of glue. After a period of thorough mixing, the students are left with a puttylike substance commonly referred to as slime. The students spend some time determining the properties of their substance and recording their observations. Mr. Shope asks students to determine whether they created a mixture or a new substance and to support their claim with evidence from their observations. Which student made the best use of evidence to support his or her claim? A.Alyssa, who wrote that they had created a new substance because the slime has properties that are different from the properties of the three ingredients B.Max, who wrote that they had created a mixture because all three ingredients were mixed together C.DeShawn, who wrote that they had created both a mixture and a new substance because they mixed the ingredients together and that by mixing them in water, they could not get the ingredients back again D.Alessandro, who wrote that they had created a mixture because the original ingredients could be brought back to their original form

Option (A) is correct. This question tests your ability to critique students' work on the basis of how well the students used evidence to support a scientific claim. The student described in (A) correctly cited the properties of the slime that are different from those of the original ingredients as evidence that a new substance had been formed in the activity.

Ms. Brandt is a fourth-grade teacher who is about to start a unit on the physical geography of her state. The content standard requires that students be able to explain changes in physical characteristics over time caused by both human and natural processes. Which of the following should Ms. Brandt consider in her lesson planning to ensure students are able to understand the concept? A.The use of essential questions that are related to the topic B.The amount of understanding her students have about ancient history C.The developmental reasoning abilities of her students D.The ability of her students to accurately complete activities that sequence events in time

Option (A) is correct. To best capture students' attention, teachers ask essential questions related to the topic when planning lessons.

After experiencing the shade from the leafy branches of a large oak tree on the school playground, Ms. Jenkins' kindergartners work together to design and build a model structure that will reduce the warming effect of sunlight on an area. Which of the following sets of materials is most appropriate for kindergartners to successfully engage in and complete these tasks? A.A few sheets of blank paper and crayons B.A bag of materials such as craft sticks, construction paper, glue, tape, and clay C.A box with scrap wood, nails, roof shingles, hammers, staples, and staple guns D.A collection of pictures showing children playing under an umbrella on the beach and on a covered porch

Option (B) is correct. A bag of such materials will provide students working in teams the supplies needed to build a structure that will reduce the warming effect of the Sun on an area.

A fifth-grade teacher is teaching a unit on explorers of North America and wants students to identify and recognize the accomplishments of prominent leaders. Which of the following is the most effective way to meet this objective? A.Showing a map highlighting the routes the explorers traveled B.Planning a jigsaw activity in which specific explorers are assigned to each group C.Providing activity packets with a story on each explorer to read at home D.Creating learning centers with information on the political conditions of that time

Option (B) is correct. A jigsaw approach allows students to work in small groups to specialize in one area, research the topic, and then teach other students what they've learned. Students will meet this objective in a more cooperative learning environment instead of simple memorization of facts.

Which of the following is a change made by the federal government as a result of the attacks on the United States on September 11, 2001, that affects how people travel? A.Taking control of the airline industry B.Increasing security measures in airports C.Creating new immigration quotas D.Requiring passport identification for domestic railway passengers

Option (B) is correct. Airport security increased after September 11, 2001, resulting in stricter random screening.

A third-grade student remarked, "I can't go to the movies with friends because I just spent all of my birthday money on a new video game." Which of the following economic concepts does this statement best illustrate? A.Barter B.Opportunity cost C.Capital resources D.Interest

Option (B) is correct. An opportunity cost is something a person gives up in order to gain something else. In this case, the student gave up going to the movies in order to gain a new video game.

Mr. Walters asked his students to order 89, 708, 37, and 93 from least to greatest, and to be ready to explain the process they used to order the numbers. One student, Brianna, ordered the numbers correctly, and when Mr. Walters asked her to explain her process, she said, "The numbers 89, 37, and 93 are less than 100, so they are all less than 708, since that is greater than 100. Also, 37 is the least because it comes before 50 and the other two numbers are close to 100. Then 89 is less than 90, but 93 is greater than 90." Which of the following best describes the strategy on which Brianna's explanation is based? A.A counting strategy B.A benchmarking strategy C.An estimation strategy D.A place-value strategy

Option (B) is correct. Brianna first indicates that 708 is the greatest number because it is greater than 100, while 37, 89, and 93 are all less than 100. Next, Brianna indicates that 37 is the least number because it is less than 50, while 89 and 93 are greater than 50. Finally, Brianna recognizes that 89 is less than 93 because 89 is less than 90, while 93 is greater than 90. Thus, over the course of her explanation, Brianna used 100, then 50, and then 90 as points of reference for comparisons, which is exactly what benchmark numbers are—points of reference for comparison. Brianna did not count between any of the numbers, estimate the numbers, or use the place values in any of the numbers to make her comparisons, so the other options do not describe the strategy on which Brianna's explanation is based.

Which of the following is an appropriate approach to teach a third-grade student about the reason for the addition of the Bill of Rights to the Constitution of the United States? A.Listing why the colonies wanted independence B.Discussing why it is important to protect individual liberties C.Comparing how the colonies were founded D.Illustrating how a national church would unite the country

Option (B) is correct. It is important for students to know that the Bill of Rights was created to protect individual rights by limiting government authority.

Ms. Roderick asked her lunch helper in her kindergarten class to get one paper plate for each student in the class. Which of the following counting tasks assesses the same mathematical counting work as this task? A.Having students line up according to the number of the day of the month in which they were born B.Showing students 10 pencils and asking them to get enough erasers for all the pencils C.Showing students a row of 12 buttons and asking them to make a pile of 8 buttons D.Asking students to count the number of triangles printed on the classroom rug

Option (B) is correct. Getting one paper plate for each student in the class assesses whether students can determine when the number of objects in one set is equal to the number of objects in another set, and the task described in option (B) involves a similar determination. The task in option (A) assesses whether students can compare and order numbers. The task in option (C) assesses whether students can count a subset of objects from a larger set. The task in option (D) assesses whether students can count the number of objects in a set.

Ms. Simeone is working with her first-grade students on writing two-digit numerals. She wants to use an activity to assess whether her students are attending to the left-to-right directionality of the number system. Which of the following activities is best aligned with Ms. Simeone's purpose? A.Asking students to read the numbers 20 through 29 B.Asking students to represent the numbers 35 and 53 using base-ten blocks C.Asking students how many tens and how many ones are in the number 33 D.Showing students 23 cubes and 32 cubes and asking them which quantity is greater

Option (B) is correct. Having the students represent 35 and 53 using base-ten blocks will help Ms. Simeone assess whether students know which place is the tens place and which place is the ones place or whether students have reversed the ones place and the tens place, thinking the ones place is on the left and the tens place is on the right. Representing the numbers provides more information about students' understanding of place value than just reading numbers.

In word problems that have a multiplicative comparison problem structure, two different sets are compared, and one of the sets consists of multiple copies of the other set. Which of the following best illustrates a word problem that has a multiplicative comparison problem structure? A.There are 4 shelves in Joaquin's bookcase, and there are 28 books on each shelf. How many books are in Joaquin's bookcase? B.Marcus drives 3 times as many miles to get to work as Hannah does. Hannah drives 16 miles to get to work. How many miles does Marcus drive to get to work? C.A football field is 360 feet long and 160 feet wide. A soccer field is 300 feet long and 150 feet wide. The area of the football field is how many square feet greater than the area of the soccer field? D.An ice cream parlor sells 29 different flavors of ice cream and 4 different types of cones. How many different combinations consisting of an ice cream flavor and a type of cone are available at the ice cream parlor?

Option (B) is correct. In the problem in option (B), the two values being compared are the number of miles that Marcus drives to get to work and the number of miles that Hannah drives to get to work, and the number of miles that Marcus drives is 3 times the number of miles that Hannah drives. The problem in option (A) has an equal-groups problem structure, the problem in option (C) has a product-of-measures problem structure (since the product is a different type of unit from the factors in the problem), and the problem in option (D) has a combinations problem structure.

Students in an upper elementary classroom are studying fossils. Which resource would best support the idea that fossils provide evidence that ecosystems can change over time? A.Actual samples of fern fossils found in a swamp B.Pictures of sea shell fossils found in a desert C.A map showing locations in a forest where scientists have discovered wooly mammoth fossils D.A chart listing the number of extinct shark teeth found at different shorelines around the world

Option (B) is correct. Marine life cannot exist in a desert. Showing students pictures will enable them to see that the shells look out of place and that the desert environment must have been a sea many years ago.

The following activity was intended to demonstrate that matter is conserved during a change of state. A student weighed an empty glass beaker and then filled it with water before weighing it again so that she could find the weight of the water. She then poured the water from the beaker into an ice-cube tray and placed it in a freezer. After the water had frozen, the ice cubes were removed from the tray and weighed. This was repeated five times. Each time the weight of the ice cubes was slightly less than the initial weight of the water. The student concluded that freezing the water reduced its mass. Which of the following procedure modifications would best remove the error that affected the student's data? A.Removing the water from the beaker and the ice cubes from the tray using a spatula B.Weighing the water after it is placed in the tray and the ice cubes before they are removed from the tray C.Repeating the experiment three times instead of five times D.Drying the beaker and ice-cube tray with a paper towel before water is added

Option (B) is correct. Modifying the procedure by weighing the water after it is placed in the tray and weighing the ice cubes before they are removed from the tray would most likely eliminate the error. Transferring the water twice for weighing probably led to a systematic loss of mass.

A sixth-grade teacher asks students to complete the following assignment. Design a political cartoon to show the system of checks and balances outlined in the United States Constitution. Write a paragraph explaining the methods and techniques used to convey the political message. To successfully complete the assignment, which of the following is important for the students to know in addition to knowing how to create a political cartoon? A.Specific examples of when presidential pardons were granted or denied B.How each branch of government limits the powers of other branches C.Knowledge of how other countries restrict the powers of government D.Names of current leaders of the three branches of government

Option (B) is correct. Students cannot visualize the checks-and-balances system without knowing ways that each branch of government can limit the powers of other branches of government.

A fourth-grade teacher introduces a civics unit on the rights and responsibilities of citizenship with the following activity. Posts signs with the words RIGHTS and DUTIES at the opposite walls of the room. Flashes photos on the classroom screen of citizens enjoying rights and completing civic duties. As each photo is projected, students walk to the sign that best describes the action being shown. Which of the following is the main purpose of the activity? A.Encouraging students to ask questions about the topic B.Determining students' prior knowledge before instruction C.Helping students participate in collaborative learning D.Involving students in critical thinking skills on the topic

Option (B) is correct. The activity informs the teacher of what students already know about the rights and duties of citizenship.

The scenario in a word problem states that an office supply store sells pens in packages of 12 and pencils in packages of 20. Which of the following questions about the scenario involves finding a common multiple of 12 and 20 ? A.In one package each of pens and pencils, what is the ratio of pens to pencils? B.How many packages of pens and how many packages of pencils are needed to have the same number of pens as pencils? C.If the store sells 4 packages each of pens and pencils, what is the total number of pens and pencils sold in the packages altogether? D.How many gift sets can be made from one package each of pens and pencils if there are the same number of pens in each set, the same number of pencils in each set, and all the pens and pencils are used?

Option (B) is correct. The least common multiple of 12 and 20 is 60, and 5 packages of pens and 3 packages of pencils are needed to have 60 of each writing utensil. The question in option (A) uses factors, not multiples, since 12 pens/20 pencils=3/5. The question in option (C) is best answered by calculating 4×12+4×20=48+80=1284×12+4×20=48+80=128, which does not involve finding either a common factor or a common multiple of 12 and 20. The question in option (D) is best answered by finding that the greatest common factor of 12 and 20 is 4, which means that 4 gift sets can be made, each containing 3 pens and 5 pencils.

Mr. Jones is teaching his class a unit on energy conversions between mechanical, electrical, and light energy. Which of the following demonstrations would best provide the class with an example of a process that converts one of these energy types into another of these types? A.Lifting a heavy box with a mechanical lever B.Opening and closing a switch in a circuit containing a battery and a lightbulb C.Producing sound by striking a drum with a stick D.Placing a closed container of cool water in a larger container of hot water

Option (B) is correct. This demonstration shows electrical energy being converted to thermal and light energy.

A history teacher assigns a short writing prompt assessing student knowledge of the impact of women such as Elizabeth Cady Stanton and Susan B. Anthony on the abolition movement. Many students submitted responses that misinterpreted the question, often stating that, "Americans learned that women should be able to vote in elections." Previous knowledge of which of the following historical developments most likely led students to misinterpret the writing prompt? A.Women were not granted the right to vote until 1920. B.Many abolitionist leaders were also leaders in the woman suffrage movement. C.Women were often responsible for the education of their children. D.Many abolitionist leaders supported the passage of the Fifteenth Amendment.

Option (B) is correct. The most likely historical development to contribute to the students' misinterpretation is the relationship between the abolition and suffrage movements. Many reformers involved in the woman suffrage movement, such as Susan B. Anthony and Elizabeth Cady Stanton, also sought to abolish slavery in the United States.

Mr. Sen is creating a lesson for his fourth-grade class around the geographic theme of human-environment interaction, which deals with the ways in which geographic locations influence the lives and choices of the people who live in them. Which of the following learning objectives best demonstrates Mr. Sen's understanding of the social studies concept of human-environment interaction? A.Students will be able to write an essay about the limitations geography placed on early explorers in the Americas. B.Students will be able to construct a table describing how different physical features of regions influence architecture and economic activities of residents. C.Students will be able to create a cause-and-effect diagram describing the formation of different landforms. D.Students will be able to describe patterns of immigration, emigration, and distribution of people in a particular region.

Option (B) is correct. The objective is most closely related to the theme of human-environment interaction because it links physical features of regions and how those features influence human activities.

Which of the following pairs of words contains identical onsets? A.Scrape/skirt B.Slip/slime C.Sand/stand D.Sting/sing

Option (B) is correct. The onset of a word is the initial phonological unit, and in the words "slip" and "slime," the onset of "sl" is the same.

The following is an excerpt from a student's draft of a short story. Once opon a time it was Holloween and Mr. Brisson and some of his studints saw a honted house. Mr. Brisson got scared. Will said Don't be scared. You are a grownop you soodent be scared. They saw the gost of Abrham Licin and smaced the door closed. Mr. Brisson got even more scared. His teeth were caddering. Based on the excerpt, the student would benefit from further instruction in which of the following areas of spelling? A.Consonant blends B.Consonant digraphs C.Common affixes D.Vowel, consonant, e (VCE)

Option (B) is correct. The student incorrectly spells a number of consonant digraphs, including the "sh" in "soodent" (shouldn't), the "gh" in "gost" (ghost), the "ck" in "smaced" (smacked), and the "ch" in "caddering" (chattering).

The students are looking at a profile of a local canyon wall. Sandy says, "I didn't realize this area was once an ocean." The teacher asks the other students to explain why they think Sandy may have come to this conclusion. Which student response indicates a lack of understanding about the evidence and geologic history? A."I can see a layer of rock with all kinds of shells in it." B."The layers of rock are in different colors in the shape of an ocean wave." C."Two of the layers are sand, with a layer of shells between them." D."The fossils we can see are from species that lived in the ocean."

Option (B) is correct. The student lacks an understanding of the evidence because the appearance of different colored layers of rocks is not evidence that the area was once covered by an ocean.

A student evaluates primary and secondary resources to gain insight into a historical topic. Which of the following will most likely lead to the student eliminating the source being consulted? A.The student observes that the diary of a person directly affected by the event recalls incorrect dates compared to other sources. B.The student notes that a published thesis on a university Web site lacks citations for the sources used to create the argument. C.The student gathers population data and statistics from a government website ending in ".gov".dot g o v D.The student reads the preface of a published book and concludes that its author takes a debatable position on the historical issues.

Option (B) is correct. The student who encounters a secondary source of this type that fails to reveal where the information came from should disregard the source and label it invalid.

Lower-elementary students are asked to use small building blocks to create a model town on a hill along with a brick road leading to a river. The teacher then asks the students to build a bridge across the river by reusing blocks from the town and road. Next, the teacher has students explain how they made the bridge. Which of the following student responses lends the best evidence that students built the bridge according to the constraints laid out by the teacher? A."We used a sheet of construction paper to serve as the bridge." B."We made the road narrower and used those bricks so now you can get across the river." C."We used some leftover bricks to build a dam above the city." D."We made some buildings shorter and used those bricks so now the road is wider."

Option (B) is correct. The students have reused material to create a bridge as indicated in the constraints laid out by the teacher.

In an introductory lesson on the United States federal government, a second-grade teacher asks the students to compare the jobs of the school's principal and vice principal to the jobs of the president and vice president of the United States. The lesson best introduces which of the following concepts about the United States government? A.The three branches of the federal government B.The role of the executive branch of government C.The process by which elected representatives create laws D.The system of checks and balances

Option (B) is correct. The teacher is asking students to relate the jobs of the school principals to the jobs of the president and vice president of the United States in order to help them develop a basic understanding of the roles and responsibilities of executive offices.

Which of the following lists contains words that can all be broken into distinct morphemes? A.People, water, gentle, pencils B.Unable, kids, playing, biggest C.Children, hope, happy, assertion D.Phone, believe, strengthen, ocean

Option (B) is correct. The words in the list can all be broken down into the following morphemes: un/able, kid/s, play/ing, big/est.

Onset

the initial phonological unit of any word (e.g. c in cat)

A first-grade teacher sets up a pretend market in the classroom. The market includes seven apples, priced at $0.50 each, and two oranges, priced at $2.00 each. The teacher explains that the price of the oranges is higher because there are fewer of them available. This lesson best illustrates which of the following concepts? A.Showing how to shop on a budget B.Demonstrating how supply affects prices C.Introducing how the climate of a region dictates the supply of produce D.Encouraging the students to save more to increase purchasing power

Option (B) is correct. This lesson illustrates how supply affects prices. All other factors being equal, items for which there is a lower supply are priced at a higher rate per item than items for which there is a more ample supply.

A student presents a project to the class describing the relationship between flowering plants and bees. The student concludes the presentation by stating, "Flowers have bright colors so they can trick bees into helping them reproduce." Which of the following responses by the teacher would best help the student develop a more accurate view of the relationship between bees and flowers? A.Have you considered the role of other animals in helping plants reproduce? B.Have you considered a benefit for bees when they help flowers reproduce? C.What benefits do humans receive from this bee-flowering plant relationship? D.Do flowering plants produce more pollen when bees are present?

Option (B) is correct. This question will direct students toward a more accurate view of the relationship between bees and flowers by adding a missing component to the model and to the explanation.

1/3=3/9=6/18 1/4=4/16=3/12 Ms. White's students are working on generating equivalent fractions like the ones shown. She asks her students to write a set of instructions for how to generate equivalent fractions. One student writes, "You have to multiply the bottom and the top of the fraction by a number." Which of the following revisions most improves the student statement in terms of validity and generalizability? A.You have to multiply both denominator and numerator by the same number. B.You have to multiply both denominator and numerator by the same nonzero number. C.You have to multiply both denominator and numerator by the same whole number. D.You have to multiply both denominator and numerator by the same positive whole number.

Option (B) is correct. To generate an equivalent fraction, it is not necessary to multiply the numerator and denominator of the original fraction by a whole number, but it is necessary to multiply the numerator and the denominator by the same number and for that number to be a number other than zero. The revision in option (B) is the only sentence that restates the student conjecture, makes it valid, and generalizes it by including all fractions.

A teacher divides a class into two representative bodies. The groups work together to create a new list of classroom rules and vote on which of the rules should be adopted. The teacher does not agree with one of the rules and takes it off the list. Which of the following examples of the system of checks and balances outlined in the Constitution of the United States is the teacher modeling? A.Making laws B.Vetoing laws C.Enforcing laws D.Interpreting laws

Option (B) is correct. Vetoing laws created by Congress is a way for the president to keep the legislative branch in check or change an act of the legislative branch. By rejecting one of the rules, the teacher is vetoing it.

Which of the following learning activities best helps students understand the Founding Fathers' motivation for writing the Declaration of Independence? A.Portraying Thomas Jefferson discussing the issue of slavery with James Madison in a role-play scenario B.Writing a letter to the principal expressing why they believe the new school uniform policy is unfair C.Working with a group of peers to create a poster showing the British what life is like in the New World D.Engaging in a class discussion of England's policy on the western land and the Iroquois neutrality pacts

Option (B) is correct. Writing an opinion letter to the principal reflects the same motivation the Founding Fathers had in writing the Declaration of Independence.

Ms. Rodriguez is working with her kindergarten students to develop the skill of counting on. Which of the following tasks is best aligned with the goal of having students count on? A.The teacher gives each student a number book with a different number on each page. The students must count out and glue the same number of pictures to match the given number on each page. B.The teacher gives each student a 10-piece puzzle, disassembled with a single number written on each piece. The students must put the puzzle together with the numbers in order. C.The teacher gives each student a shuffled deck of 10 cards, each with a single number from 1 to 10. When the students draw a number card, they must count to 20, starting from the number on the card they drew. D.The teacher gives each student 8 blocks and a number cube, with the sides of the number cube numbered from 3 to 8. When the students roll the number cube, they must count out the same number of blocks as the number rolled and create a tower with that number of blocks.

Option (C) is correct. A student would begin with the number drawn and count on from that number until 20 is reached. For example, if the student draws a card with 15 on it, the student would count on from 15, saying, "15, 16, 17, 18, 19, 20." The other tasks described do not require students to count on.

A teacher has a student read the following sentence aloud. I like to go to the movies with my family. When reading the sentence, the student says, "I like to go to the moving with my family." In this case, the student is most likely relying on which of the following cuing systems while reading? Select all that apply. A.Syntactic B.Semantic C.Graphophonemic

Option (C) is correct. Graphophonemic cues are visual clues, including the ones that this student is mistakenly relying on when he says "moving" instead of "movies." The two words share some letters and a similar construction.

As an introduction to an economics lesson, a teacher uses the example of a small business that prepares to launch a new cereal in the market. The teacher emphasizes that the cereal can only be sold in stores after its ingredients are tested and the nutritional information has been verified by the local and county health agencies. The teacher's example best illustrates which of the following concepts? A.Supply and demand B.Investment C.Regulation D.Corporate taxation

Option (C) is correct. All local and county health agencies have to follow the federal regulations as set by the Food and Drug Administration (FDA)F D A. This organization places restrictions on the food businesses before they can market a product to a consumer.

Ms. Khouri holds a helium-filled balloon in one hand and a rubber ball in the other. She asks her fifth-grade students what will happen when she lets go of the objects. All of the students agree that the balloon will rise to the ceiling and the ball will fall to the ground. When Ms. Khouri asks why, one student responds that it is because the balloon is light and the ball is heavy. Which of the following demonstrations would best challenge the student's misconception that only heavy objects fall to the ground? A.Tying the ball to the helium-filled balloon with a light string and releasing them together B.Simultaneously releasing the helium-filled balloon and throwing the ball up toward the ceiling C.Simultaneously releasing a helium-filled balloon and an identical, but uninflated, balloon D.Having the students use stopwatches to time how long it takes for the balloon to reach the ceiling and for the ball to reach the ground

Option (C) is correct. By presenting the student with two light objects, one rising and one falling, the student must find another explanation.

A fourth-grade teacher wants students to understand that conducting a comparison of costs and benefits is the best method to use when selecting an item to purchase. Which of the following strategies provides the most effective way of accomplishing this goal? A.Placing four similar items on the teaching station with the same price tags and asking the students to choose one, then explain their choices B.Asking students to provide a graphical representation of marginal cost and benefit of a product as a cost-benefit analysis study C.Distributing two store fliers and having students select one item to compare the prices for at each store, then determine which store to purchase the item with an explanation of why D.Conducting an interactive lesson showing how commercials dictate people's purchasing decisions, regardless of the costs

Option (C) is correct. Comparing similar items at different stores allows students to weigh the costs and benefits of one item, as opposed to the costs and benefits of different items.

During a history unit on the American Revolution a fifth-grade teacher teaches a lesson on the complex relationships between historical actors leading to the outbreak of the revolution. Which of the following homework assignments would best assess the learning objective? A.Students create a time line of major battles during the revolution. B.Students answer multiple-choice questions identifying revolutionaries. C.Students create a concept web visualizing the interactions of revolutionaries. D.Students write a biographical essay on an assigned revolutionary.

Option (C) is correct. Creating a concept web of the interactions between revolutionaries would assess students' understanding of complex relationships between historical actors.

Which of the following assignments would best allow a fifth-grade teacher to determine whether students fully understand push and pull factors? A.Drawing a Venn diagram of the features of a fictional character's country of immigration and country of emigration B.Creating a time line of different immigration patterns to the United States C.Creating a storyboard for a documentary about the top reasons people migrate D.Listing the ways people can improve their lives after migration

Option (C) is correct. Creating a storyboard will allow students to visualize and help them remember both push and pull factors for migration.

A first-grade teacher provides students with several pictures of daily school day activities, such as coming into the classroom in the morning, doing schoolwork, playing outside during recess, and boarding a school bus. The teacher asks students to place the pictures on a timeline according to when they occur in the school day. Which of the following is the primary purpose of the activity? A.Identifying commonalities among events B.Prioritizing tasks on a list according to importance C.Introducing the idea of placing events in a chronological order D.Comparing a school schedule now and how it was long time ago

Option (C) is correct. Early elementary social studies lessons typically introduce the concept of chronology using daily events that the students can relate to.

Students are conducting an investigation on dissolving solids in a liquid. After mixing a spoonful of salt into a large container of water, a student says, "I can no longer see the salt in the water because the salt is gone." What can the teacher do to clarify this misconception? A.Add another teaspoon of salt to the water and stir. B.Show the students that other substances also can dissolve by mixing sugar and water together. C.Evaporate the water and show that the salt still remains. D.Give the students a magnifying glass and allow them to inspect the sample.

Option (C) is correct. Evaporating the water will show that the salt still remains in the container.

To assess the students' mastery of providing explanations, the teacher of a third-grade class asks students to select a particular species that lives in groups and write an explanation of how a particular type of interaction within the group helps increase the likelihood that individuals will survive to grow and reproduce. Which student explanation best meets the objective? A."Whales live in pods. Whales migrate to warmer waters to survive." B."Bees live in colonies. Each bee has a specific duty to maintain the colony." C."Fish live in schools. By being one of a large number of fish, each individual has less chance of being eaten." D."Elephants travel in herds. The calves run slower and are surrounded by the older elephants."

Option (C) is correct. In this explanation, the student describes a specific behavior to support the claim and makes a connection to how the school provides protection for all its members.

A teacher shows students a short video of a ball being thrown vertically upward, momentarily coming to rest, and falling back down toward the ground. When asked about the force of gravity on the ball throughout its motion, a student says that gravity pulls down on the ball at all times except for when the ball stops briefly before reversing direction. Which of the following responses by the teacher will best help the student overcome the misconception about gravity? A.Showing the video again but in slow motion B.Posing a follow-up question: How does a parachute change the motion of a skydiver in free fall? C.Posing a follow-up question: If gravity is not pulling on the ball when it stops, why does it start to fall back down? D.Showing a video highlighting variations in gravity between Earth and the Moon and at different altitudes on Earth

Option (C) is correct. It will help the student overcome the misconception by knowing that an object can change direction only if there is a force acting on it. This response will show the student why gravity is still pulling on the ball even when it is momentarily at rest.

Ms. Cook's class was discussing strategies to compare two fractions. One student, Levi, said, "When the top numbers are the same, you know that the one with the smaller number on bottom is bigger." Ms. Cook asked her students to explain why Levi's claim is true. After giving the class time to work, she asked another student, Maria, to present her explanation. Maria said, "It's just like Levi said. For 1/4 and 1/2, they both have ones on top, and 4 is greater than 2, so 14 is less, just like 1/4 of a pizza is less than 1/2 of a pizza." Which of the following statements best characterizes Maria's explanation? A.It clearly explains why Levi's claim is true. B.It clearly explains why the converse of Levi's claim is true, but it does not explain why his actual claim is true. C.It shows that Levi's claim is true for one example, but it does not establish why his claim is true in general. D.It assumes that Levi's claim is true, but it does not establish why his claim is true in general.

Option (C) is correct. Maria explains why 1/4 is less than 1/2, which provides one example for which Levi's claim is true, but it does not explain why whenever two fractions have the same numerator, the fraction with the smaller denominator will always be the greater fraction. A general explanation would point out that when a whole is broken into a greater number of pieces of equal size, then each of those pieces will be smaller than the pieces when the whole is broken into fewer pieces of equal size.

Which of the following best allows students to compare and contrast their cultures with the cultures of others? A.Watching a documentary on the factors that are responsible for mass migration B.Reading an article in a school newspaper about how local politics influence education in different countries C.Writing and receiving letters from peer pen pals from a school in another country D.Studying immigration and emigration patterns of groups from other countries

Option (C) is correct. Organizing a classroom community where students will write and receive letters from pen pals from another classroom in another country will promote literacy and increase knowledge about other cultures.

A first-grade teacher presents a lesson on landforms and water features. When doing a follow-up activity, several students confuse an isthmus with a strait, a peninsula with a gulf, and a cape with a bay. Which of the following strategies most effectively addresses the students' misunderstanding? A.Planning a hands-on project about land forms for the students to complete at home with parental guidance B.Asking a student volunteer to explain to the class in his or her own words the differences in the land forms and water C.Presenting a world map with the land and water forms to review the newly learned information D.Allowing the students to conduct their own research on the various landforms

Option (C) is correct. Reviewing the various landforms and water features using the visual aid of a world map will help clarify the confusion the students are having.

Which of the following best exemplifies how capital can function as a resource in the process of creating goods? A.A land developer uses personal income to buy a parcel of land and to organize a construction team to build a new residential property. B.A carpenter is paid for several weeks of work cutting wood and fitting it together to provide the foundation for a new home. C.A hammer created by a tool manufacturer is used by a construction worker in the process of building a new residential property. D.A logger is placed in charge of cutting trees in a forest that are later refined into wood used to create structures for new homes.

Option (C) is correct. The hammer functions as capital rather than a consumer good and is a resource that is being utilized to create a new good, in this case a residential property.

Ms. Duchamp asked her students to write explanations of how they found the answer to the problem 24×15. One student, Sergio, wrote, "I did 24 times 10 and got 240, then I did 24 times 5 and that's the same as 12 times 10 or 120, and then I put together 240 and 120 and got 360." Ms. Duchamp noticed that four other students found the same answer to the problem but explained their strategies differently. Which of the following student explanations uses reasoning that is most mathematically similar to Sergio's reasoning? A.Since 24 is the same as 12 times 2 and 15 is the same as 5 times 3, I did 12 times 5 and got 60, then I did 2 times 3 and got 6, and 60 times 6 is 360. B.To get 24 times 5, I did 20 times 5 and 4 times 5, which is 120 altogether, and then I needed 3 of that, and 120 times 3 is 360. C.15 times 20 is the same as 30 times 10, and that gave me 300, and then I did 15 times 4 to get 60, and 300 plus 60 is 360. D.24 divided by 2 is 12, and 15 times 2 is 30, so 24 times 15 is the same as 12 times 30, and so my answer is 360.

Option (C) is correct. Sergio first uses the distributive property to think of 24×15 as 24×(10+5), or 24×10+24×5. After Sergio multiplies 24 and 10 to get 240, he multiplies 24 and 5 using a doubling and halving strategy. Since 24=12×2, 24×5=(12×2)×5=12×(2×5)=12×10, so the product of 24 and 5 is equal to the product of 12 and 10, which is 120. The explanation in option (C) also uses the distributive property but in a different way. This student thinks of 24×15 as (20+4)×15, or 20×15+4×15. After the student multiplies 20 and 15, the student uses the doubling and halving strategy to find the product of 4 and 15. Therefore, this explanation uses reasoning that is most mathematically similar to Sergio's reasoning. The explanations in options (A) and (B) do not use the doubling and halving strategy, and the explanation in option (D) does not use the distributive property.

As a part of a lesson, a fourth-grade class identifies community issues and the process by which local government addresses those needs. Which of the following student activities is most likely to develop an understanding of the roles and responsibilities of the local government? A.Brainstorming interview questions to ask a community leader B.Discussing possible solutions to current political issues in the country C.Participating in a simulation of a city council meeting to address local issues D.Conducting a mock election for various local government positions

Option (C) is correct. Simulating a city council meeting requires students to discuss issues, assume roles and responsibilities of local leaders, and brainstorm solutions in a class setting. The activity is hands-on and is the most effective way to learn in social studies.

Which of the following fractions has a value between the values of the fractions 7979 and 811811 ? A. 1/2 B. 2/3 C. 3/4 D. 4/5

Option (C) is correct. Since 7/9≈0.78 and 8/11≈0.73, the fraction 34 has a value between the values of 7/9 and 8/11 because 3/4=0.75.

A second-grade teacher introduces a civics activity in which students sort cards into two different categories. The cards are each labeled with activities such as voting in elections, celebrating a religious holiday, volunteering at homeless shelters, and protesting an unfair law. Which of the following best describes the objective of the activity? A.Creating classroom rules collectively B.Reviewing ways students can participate in society C.Identifying the rights and responsibilities of living in a democracy D.Analyzing the effectiveness of rules and laws

Option (C) is correct. The activity requires students to understand the duties of a citizen and to distinguish between rights and responsibilities by sorting labeled cards.

A teacher prepares a lesson to demonstrate how individual perspectives alter the ways in which people react to the same event. The historical focus for the lesson is the Boston Massacre. Which of the following pairs of resources best facilitates the teacher's overall objective? A.A current history magazine article and a newspaper article written in 1770 B.A modern encyclopedia entry and an article published in a scholarly journal C.A diary entry from an American colonist and court testimony of a British officer D.A court transcript from an American colonist and a section from a history textbook

Option (C) is correct. The court testimony and the diary entry both qualify as primary sources and as such offer the best evidence for how the opposing sides viewed the conflict from their unique lenses.

A history teacher wants to use a documentary video to engage students in a classroom discussion of the ways that pre-Columbian civilizations in Central and South America were influenced by their natural environments. Which of the following documentary videos would best facilitate the discussion? A.A documentary about the similarities among pre-Columbian languages B.A documentary about Spanish Conquest of the Aztec civilization C.A documentary about the development of agriculture in different pre-Columbian civilizations D.A documentary video about species of wildlife that live in Central America

Option (C) is correct. The documentary video about pre-Columbian farming techniques would best facilitate a discussion of how pre-Columbian civilizations were influenced by their natural environment because students will learn what kinds of crops were available in each region, and what methods were best suited to the environments they lived in.

Ms. Chamberlain's students are discussing the following quadrilaterals and their diagonals. "The figure presents 3 quadrilaterals divided into sections." Ms. Chamberlain asks her students what they notice about the diagonals in the quadrilaterals. One student says, "I noticed that the diagonals of the quadrilaterals always cross at a right angle." Of the following sets of quadrilaterals, for which set is the student's conjecture always true? A.Quadrilaterals with one pair of congruent opposite angles B.Quadrilaterals with one pair of parallel opposite sides C.Quadrilaterals with two pairs of congruent adjacent sides D.Quadrilaterals with two pairs of parallel opposite sides

Option (C) is correct. The quadrilaterals Ms. Chamberlain's students are discussing are, from left to right, a square, a rhombus, and a kite. All kites have perpendicular diagonals, and kites are defined as quadrilaterals with two pairs of congruent adjacent sides. Squares and rhombuses have two pairs of congruent adjacent sides as well, since they are kites with additional restrictions on their characteristics. Quadrilaterals with one pair of congruent opposite angles could be scalene quadrilaterals, which do not necessarily have perpendicular diagonals, so option (A) is incorrect. Quadrilaterals with one pair of parallel opposite sides are trapezoids, which do not necessarily have perpendicular diagonals, so option (B) is incorrect. Quadrilaterals with two pairs of opposite parallel sides are parallelograms, which do not necessarily have perpendicular diagonals, so option (D) is incorrect.

Upper-elementary students were asked to interpret the results of a study carried out by a team of paleontologists. The students were then asked to formulate a scientific explanation of the paleontologists' observations. In which of the following responses does the student best use the paleontologists' observations to support a claim about the history of Earth? A."The paleontologists first found a layer of plant fossils covered by many rock layers. On top of that were land fossils. Then there were more rock layers." B."The paleontologists found some patterns in the rocks. They first found land fossils and then they found plant fossils. Then they found marine fossils. This shows that the land was dry and then it had been covered by water." C."The paleontologists found a rock layer containing shells and fish below a rock layer containing fossils of land animals and plants. This shows that the land had been covered by water and later it was dry land." D."The paleontologists found marine fossils and plant fossils in the rock layers. This means the land was once covered by water and once it was dry."

Option (C) is correct. The student explains the core idea that the landscape had been covered by water at one point and that later it became dry land. The explanation is correct and specifies the sequence of events.

During a class demonstration, a teacher floats a small toy boat in the center of a long, narrow rectangular tank filled with water. Initially, the water is still and the boat is not moving. A student is instructed to predict what will happen if he uses his hand to create a wave crest that travels toward the other end of the tank, as shown in the figure. The student predicts the boat will be carried by the wave to the other end of the tank. Which of the following statements indicates a misconception that the student most likely has about water waves? A.Water waves get smaller as they get further away. B.Water waves transfer energy from one location to another. C.Water particles at the surface move along the surface with the wave. D.Water waves slow down when they encounter an obstacle.

Option (C) is correct. This statement indicates a misconception that water moves along with waves and will take the boat with it.

A first-grade teacher is beginning a unit on the predictable pattern of the Sun as it relates to seasonal changes. The goal of the unit is to teach students that as the days get longer, Earth heats up, and as the days get shorter, Earth cools down. Which of the following activities best supports the instructional goal of the unit? A.Looking at photos of the Sun's position at different times of the day and predicting Earth's movement B.Completing a Venn diagram to compare and contrast temperatures of substances at sunrise and sunset C.Studying a graph that tracks sunrise, sunset, and corresponding temperatures for each month to determine a trend D.Sharing personal experiences of observing sunrise and sunset in different seasons

Option (C) is correct. Tracking sunrise, sunset, and daily temperatures on a graph over a period of time should help students link temperature changes with seasons. This activity also provides students with data to refer to during classroom activities throughout the unit.

Which of the following enduring understandings best helps students understand the issues that led to the creation of the constitution of the United States and the Bill of Rights? A.The United States emerged as a world power with influence which spanned the globe. B.Symbols unite communities and demonstrate citizenship. C.A democracy depends on citizens understanding and respecting their individual rights and responsibilities. D.Resources can be used to determine directions, distances, and locations.

Option (C) is correct. Understanding the circumstances that led to the creation of the Constitution will lead a student to understand that democracy is reliant on its citizens understanding their own rights and responsibilities.

During a class discussion about stars, a student claims that the Sun is the largest and brightest star in the universe. Which of the following activities is most likely to help the student appropriately compare the Sun and other stars? A.Looking at posters of the day and night skies to compare the size and brightness of the Sun and other stars B.Researching how Earth's atmosphere affects the apparent size and brightness of the Sun and other stars C.Viewing a brightly lit sphere close up and then from farther away down a long hallway D.Shining a high-power flashlight and then shining a low-power flashlight onto the classroom wall

Option (C) is correct. Viewing the sphere from two separate distances will allow students to see that a bright object will appear smaller and dimmer when viewed from farther away.

A teacher wants students to increase their knowledge about specialization and trade. Which of the following activities best allows students to reach this goal? A.Providing a graphic organizer for students to compare and contrast trade barriers between two countries B.Asking students to give an oral presentation on the economic conditions of a country that has limited resources C.Having students list names of countries that generate the highest revenue through international trade D.Assigning students a country to research how surplus products are sold to other countries that have scarcity of that product

Option (D) is correct. Focused research on a country will provide in-depth knowledge on specialization and trade and will help the students to understand the dependencies among countries.

Mr. Poynter asked his students to calculate the product 0.6×0.05 by converting the decimals into base-ten fractions. One student, Larissa, answered the problem as represented in the work shown. "The figure presents 4 lines of Larissa's work while answering the problem." When Mr. Poynter asked Larissa to explain her strategy, she said, "I wrote the decimals 6 tenths and 5 hundredths as fractions and multiplied them. 6 times 5 is 30 and 10 times 100 is 1,000, so I got 30 thousandths. I cancelled out a zero on the numerator and a zero on the denominator and got 3 hundredths." Which of the following changes to Larissa's explanation is best for clarifying the mathematics that underlies her strategy? A.She should indicate why 0.6=6/10 and why 0.05=5/100. B.She should indicate why 6/10×5/100=30/1000. C.She should point out that 30÷10/1000÷10=3/100. D.She should point out that 0.03=3/100.

Option (C) is correct. When Larissa said that, for 30/1000, she "cancelled out a zero on the numerator and a zero on the denominator and got 3 hundredths," the mathematics that underlies why 30/1000=3/100 is not clear. For example, could this thinking be extended to conclude that 22/32=2/3? For her explanation to be clearer, she should have indicated that she divided both the numerator, 30, and the denominator, 1000, by 10 to obtain the equivalent fraction 3/100.

A student, Sandy, seems to love reading and reads very quickly. When reading individually, she is usually the first one finished. During read-alouds, she volunteers often and enthusiastically, though the other students struggle to understand her because she reads so fast. Her teacher has noticed, however, that Sandy has difficulty answering questions about what she has read. Which of the following best describes the problem Sandy is most likely having? A.She is struggling with her reading rate, which is hindering her fluency. B.She is struggling with fluency, which is hindering her comprehension. C.She is struggling with monitoring her comprehension while reading. D.She is struggling with prosody and accuracy while reading.

Option (C) is correct. While Sandy can read fluently and quickly, she "has difficulty answering questions about what she has read," indicating a likelihood that she is neglecting to periodically stop to assess or monitor her own comprehension.

One of Mr. Terry's students, Yvonne, found the answer to the problem 38×29 as represented in the work shown. "3 eighths times 2 ninths, equals, 2 eighths times 3 ninths, which equals 1 fourth times 1 third, which equals, 1 twelfth" When Mr. Terry asked Yvonne to explain her work, she said, "It's easier to just switch the numerators to make simpler fractions." Which of the following statements is true of Yvonne's strategy? A.Yvonne's strategy can only be used to rewrite products of fractions where the difference between the numerators is 1 and the difference between the denominators is 1. B.Yvonne's strategy can only be used to rewrite products of fractions where both fractions are less than 1. C.Yvonne's strategy can be used to rewrite any product of two fractions, but it will not always result in fractions that can be simplified. D.Yvonne's strategy can be used to rewrite any product of two fractions, and it will always result in fractions that can be simplified.

Option (C) is correct. Yvonne's strategy is valid because it is based on the definition of the product of two fractions and the commutative property of multiplication. For any two fractions ab and cd, the product of the fractions is defined as a/b×c/d=a×c/b×d. Based on the commutative property of multiplication, a×c/b×d=c×a/b×d, which means that a/b×c/d=c/b×a/d. However, Yvonne's strategy does not always result in fractions that can be simplified. For example, 4/5×6/7 is equivalent to 6/5×4/7, but none of these fractions can be simplified, and the product 24/35 cannot be simplified either.

A sixth-grade class is studying democracy. The teacher identifies that most of the students have a misunderstanding that all governments that conduct elections are democracies. Which of the following activities best addresses the misperception? A.Assigning a reading that evaluates the current voter turnout rate in countries across the world B.Asking students to analyze political cartoons satirizing corruption in an election C.Assigning students to compare voter turnout statistics for different age groups in the last election D.Conducting a mock election where only one student is nominated and placed on a ballot for each class office

Option (D) is correct. A mock election with only one candidate shows students that simply having an election and voting do not make a democracy.

Ms. Shaughnessy is working with her class on measuring area using nonstandard units. While the students are finding the area of the surface of their desks using rectangular note cards, one student says, "I can just measure the long side of the desk with the long side of the card, then measure the short side of the desk with the short side of the card, and multiply them." Which of the following best describes the validity of the student's strategy? A.The strategy is not valid because the same unit must be used to measure each side of the desk. B.The strategy is valid only if the note cards are squares. C.The strategy is valid and the unit of measurement is square units. D.The strategy is valid and the unit of measurement is note cards.

Option (D) is correct. Area can be measured using any two-dimensional unit that covers a surface, but the label of the area must reflect that unit. In this case the student has used note cards as the unit to measure the area of the desk. When using square units, one counts how many times the side of the square unit fits on each side of the rectangle whose area is to be measured. When using a unit that is not a square, like a note card, it is important to keep the orientation of the unit constant to cover the area without overlapping. This method results in one dimension of the rectangle being measured with the long side of the note card and the other dimension of the rectangle being measured with the short side of the note card.

Ms. White's students are investigating why some stars appear to be brighter than others in the night sky. They take observational notes about the apparent brightness of two different-sized flashlights shining from various distances onto a wall in the front of the room. After the investigation, Bobby states his conclusion that stars that appear brighter in the night sky must be larger than stars that appear dimmer. Which activity will provide the best data for directing Bobby toward a more accurate conclusion? A.Repeating the investigation, aiming the flashlights at black construction paper taped to the wall B.Combining the observations made by all members of the class C.Repeating the investigation, measuring the diameter of the circles of light produced at various distances D.Repeating the investigation, using two flashlights of the same size

Option (D) is correct. Bobby focused on the size of the flashlights rather than the change in apparent brightness as the distance changed. If the flashlights are the same size, that variable will be removed and he will see that the apparent brightness changes with the distance of the flashlight.

A fifth-grade teacher asks students to create group presentations about the contributions of a classical civilization. One group wants to do their project on ancient Rome. They plan to include the Coliseum, the Pantheon, aqueducts, and the Forum. They also want to wear Roman clothing while presenting to the class. Which of the following questions should the teacher ask the group to best help the students prevent factual errors? A.How do you plan to divide the work among the members of the group? B.Are you going to create the costumes or buy them? C.How do you plan to present the information to the class? D.What sources are you planning to consider for your research?

Option (D) is correct. If the students are either not researching their topic or not using valid sources, the teacher will need to guide the students to adequate information on their topics.

Mr. Aronson noticed that one of his students, Wesley, incorrectly solved a multiplication problem, as represented in the work shown. "The figure presents the work Wesley did to multiply 34 times 52. The problem is written vertically with 34 on top and 52 on bottom. There is a small 2 above the 3 in 34. Below the 52 is a line to signify "equals." Below the line is the number 68. Below the 68 is the number 170 aligned so that the place values in 68 and 170 line up on the decimal. Below the 170 is another line to signify "equals," and below that is the number 238 aligned so that its place values line up with the place values in the numbers above." Which of the following most likely describes the reason for Wesley's error? A.Wesley did not regroup correctly. B.Wesley did not add the regrouped 2 correctly. C.Wesley did not apply his multiplication facts correctly. D.Wesley did not attend to the place value of each digit in the factors correctly.

Option (D) is correct. In his work, Wesley correctly multiplied 34×2 to get 68 for the first partial product, but in the second partial product, he did not attend to the place value of the 5 and calculated 34×5 instead of 34×50.

Ms. Carter shows one of her students, Brandon, a set of cubes. She tells Brandon that there are 13 cubes in the set and asks him to take 1 cube away from the set. Ms. Carter then asks Brandon, "How many cubes do you think are in the set now?" Brandon quickly answers, "Twelve." Brandon has demonstrated evidence of understanding which of the following mathematical ideas or skills? A.Using numerals to describe quantities B.Counting with one-to-one correspondence C.Recognizing a small quantity without counting D.Knowing that each previous number name refers to a quantity which is one less

Option (D) is correct. In the scenario, Ms. Carter shows Brandon a set of cubes, explicitly tells him how many cubes are in the set, and asks him to take one cube away from the set. This process allows Ms. Carter to ensure that Brandon knows that there is now one less cube in the set. When Ms. Carter asks how many cubes are in the set after one cube is removed, Brandon readily states, without counting the cubes, that there are 12 cubes. This provides evidence that Brandon knows that 12 is the number name that precedes 13 and that 12 refers to a quantity that is one less than 13; it can also be assumed that Brandon has the same understanding for other whole numbers. Brandon did not use written numerals in the scenario, so option (A) is not correct. Also, Brandon is told how many cubes are in the set, so there is no evidence that he can count with one-to-one correspondence or recognize a small quantity without counting, so options (B) and (C) are not correct.

A student incorrectly answered the problem 305.74×100. The student's answer is represented in the work shown. 305 point 74 times 100 equals 305 point 7 4 0 0 Which of the following student work samples shows incorrect work that is most similar to the preceding work? A.246 point 7 times 100, equals 2467 B.13 point 0 5 times 100, equals 13500 C.46 point 1 3 times 10 equals 460 point 1 3 0 D.94 point 0 3 times 10 equals 94 point 0 3 0

Option (D) is correct. In the work shown, when the student multiplied 305.74 by 100, the student rewrote 305.74 and added two zeros at the end. The work sample that is most similar to this is the sample in option (D), since this sample shows that when the student multiplied 94.03 by 10, the student rewrote 94.03 and added one zero at the end.

Ms. Fisher's students are working on identifying like terms in algebraic expressions. When Ms. Fisher asks them how they know when terms are like terms, one student, Coleman, says, "Like terms have to have the same variable in them." Ms. Fisher wants to use a pair of terms to show Coleman that his description of like terms is incomplete and needs to be refined. Which of the following pairs of terms is best for Ms. Fisher to use for this purpose?

Option (D) is correct. The best pair of terms for Ms. Fisher's purpose should contain the same variable but should not be like terms. The only option that shows such a pair is option (D), in which the variables are the same but the terms are not like terms because they have different exponents.

An elementary school class has been learning about how animals can change the environment to meet their needs. After viewing a video of a beaver building a dam, the students are asked to explain how changing its environment helps the beaver to support its needs. Their explanation needs to be supported by evidence. Which explanation responds to the teacher's direction and is most likely to be supported by evidence from a video about beavers? A."When the dam is built, the flow of water in the river changes, and a lake is formed." B."Dams are good for beavers, and beavers that build dams are better off than beavers without dams." C."Beavers are very good swimmers and can stay underwater for long periods of time." D."Because the beaver lives in a lodge in the lake formed by the dam, it is protected from predators."

Option (D) is correct. In this explanation, the student provides a claim and links the claim to a scientific reason.

A second-grade teacher is explaining how geographic events like earthquakes, hurricanes, and volcanic eruptions happen repeatedly in the same places. The teacher wants the students to associate these geographic events with changes that humans make in order to live near these places. Which of the following would best emphasize the theme of the lesson? A.Showing videos on the various causes of natural disasters around the world B.Looking at world maps showing where fault lines and lava domes are located C.Studying a chart graphing the frequency of geographic events in various places over the last ten years D.Looking at pictures of communities located near fault lines, coastal regions, and lava plateaus

Option (D) is correct. Pictures of communities in areas prone to geographic events are likely to show the modifications that humans have made to the areas in order to live there safely (e.g., sand dunes in coastal regions).

Which of the following activities would best introduce students to a method of erosion control? A.Students form mounds of soil that vary in height. After pouring water on the mounds, they compare the amount of erosion in each sample. B.Students blow through a straw on a tray of sand. They blow with varying force and compare the amount of sand that is blown away after each test. C.Students pour a different amount of water onto each of three identical trays of soil set at various slope angles, graphing the relationship between the amount of runoff and the slope angle. D.Students are presented with two trays of soil set at a slight angle, one containing growing grass. The students pour water on both samples and compare the amounts of soil lost.

Option (D) is correct. Planting grass or other plants is one of the best ways to prevent erosion. Students will notice that in the sample with vegetation, there was less soil lost.

Last Tuesday, a group of 5 researchers in a laboratory recorded observations during a 24-hour period. The day was broken into 5 nonoverlapping shifts of equal length, and each researcher recorded observations during one of the shifts. Which of the following best represents the amount of time each researcher spent recording observations last Tuesday? A.Between 4 and 4 1/4 hours B.Between 4 1/4 and 4 1/2 hours C.Between 4 1/2 and 4 3/4 hours D.Between 4 3/4 and 5 hours

Option (D) is correct. Since the 24-hour period is broken into 5 overlapping shifts of equal length, the problem is solved by finding 24/5, which is equivalent to 4 4/5. Since 4 4/5 is greater than 4 3/4 and less than 5, each researcher spent between 4 3/4 and 5 hours recording observations last Tuesday.

A second grade teacher labeled cardinal and intermediate directions in the classroom at the start of a unit of directions. Which of the following instructional strategies bests reinforces concepts from the lesson? A.Giving students worksheets of a community map that they can complete at home B.Asking students to use the labeled directions and make a list of all the possible directions C.Giving each student a compass to navigate their way around the school whenever possible D.Asking students to move to the northeastern side of the classroom using the labeled directions

Option (D) is correct. Students can refer to the cardinal direction labels in the classroom and utilize this information to support the newly learned intermediate direction information.

A second-grade teacher creates a classroom store stocked with toys that can be purchased with certain amounts of tokens. A standard amount of tokens is awarded to students each week. At the end of each week students may choose to spend their tokens on small toys from the store or save their tokens to purchase larger toys later. Understanding of which of the following economic concepts is best reinforced by the classroom store? A.Increasing productivity B.Predicting demands C.Avoiding debt D.Saving money

Option (D) is correct. Students who choose to save their money to buy larger toys from the classroom store will learn the value of saving money.

Students are asked to recreate a sign that Susan B. Anthony and Elizabeth Cady Stanton would have marched with in support of their cause. Which of the following best exemplifies a slogan that would appear on their sign? A.End corporate monopolies B.Feminism first C.Equal employment opportunities D.Let women vote

Option (D) is correct. Susan B. Anthony and Elizabeth Cady Stanton led the woman suffrage movement to help women gain the right to vote.

A second-grade teacher asks students to explain how patriotic symbols and practices reinforce national identity for Americans. Which of the following student responses demonstrates a misconception that the teacher needs to address? A.The Statue of Liberty represents America's independence and welcomes individuals coming to the United States. B.The American bald eagle often symbolizes the courage of the soldiers and individuals that created the United States of America. C.The stars on the American flag represent the 50 states that make up the United States of America. D.The Fourth of July celebration commemorates the surrender of the British army during the American Revolution.

Option (D) is correct. The Fourth of July celebration commemorates the signing of the Declaration of Independence in 1776. The British army surrendered in 1781 and the Treaty of Paris, which recognized American independence, was not signed until 1783.

Which of the following activities provides fourth-grade students with the best opportunity to make and test predictions about the transfer of kinetic energy between objects? A.Dropping a soccer ball from the roof of a building and measuring the height of the rebound B.Placing two soccer balls touching each other on the floor with both balls at rest C.Measuring the distance traveled by a soccer ball when it is allowed to roll down a ramp and onto a smooth, flat surface D.Rolling one soccer ball along the floor into another that is not moving

Option (D) is correct. The activity involves the transfer of kinetic energy between soccer balls. Because the activity is easy to observe and easy to repeat, it offers fourth-grade students an excellent opportunity to test their predictions about the transfer of kinetic energy between objects.

Which of the following examples best describes the concept of specialization in the process of producing goods? A.A medical doctor opens a practice where patients can come in to be examined for all of their health issues. B.A steel manufacturing company operates one large steel mill where all of their raw materials are processed. C.A restaurant employs one kitchen chef who is responsible for making and preparing all dishes ordered by customers. D.A company servicing the automotive industry makes only a type of tire that is compatible with pickup trucks.

Option (D) is correct. The company that manufactures only one type of tire is specializing in that product, exemplifying the efficiency that comes with specialization in the production of goods.

A teacher provides students with several rectangular samples of flexible materials, with the same length and width but varying thickness. The students are asked what causes the difference in stiffness among the materials. A student picks up a thin sample that is very flexible and a thick sample that is much stiffer. The student concludes that the thick sample must be made of a stiffer material since it is harder to bend. Which of the following sets of samples should the teacher provide to allow the student to determine that both the type of material and the thickness of the material can affect stiffness? A.Two samples of the same material of equal thickness B.Samples of the same material but of varying thickness C.Thick samples of one material that can be peeled in layers D.Samples of two different materials with two thicknesses each

Option (D) is correct. The four samples will allow the student to determine that both the thickness of the material and the type of material the sample is made of can affect stiffness.

Which of the following sentences shows a comma being used after an introductory phrase? A.Buster is a strong, healthy dog. B.No, you should not buy that shirt. C.Anna wrote the winning paper, so she received the prize. D.During the recital, many children were restless.

Option (D) is correct. The introductory phrase, "During the recital," is offset by a comma.

Which of the following classical Chinese contributions was invented to replace bamboo, wood, and silk as methods for spreading literature and information? A.Map B.Compass C.Money D.Paper

Option (D) is correct. The invention of paper helped in making books that led to the spread of literacy in a more convenient and cheaper way.

During a discussion about the moral of the fable, one student says, "Morals need to be things like 'keep trying' and 'be kind to others.' 'Let a sleeping dog lie' isn't a moral." Which of the following aspects of the story is most likely causing the student's confusion? A.The moral is stated only at the end of the story. B.The moral is not stated by the wisest character. C.The moral is demonstrated through the personification of animals. D.The moral is written in figurative language.

Option (D) is correct. The moral "Let a sleeping dog lie" includes an idiom meant to be extended to a figurative level of understanding. When a person says, "Let a sleeping dog lie," he or she rarely means to literally leave a dog alone. Rather, when used idiomatically, "Let a sleeping dog lie" implies that one should leave things alone that are likely to cause problems or that one should avoid disturbing something that does not have to be bothered. Most lower elementary students would not make this figurative leap without instruction from the teacher.

Mr. Yu presents his class with a series of pictures of four different ecosystems: grassland, tundra, desert, and tropical rain forest. He instructs the students to count the number of different kinds of plants and animals they can find in the pictures of each ecosystem. The class data is placed on the board and discussed. Which concept does the activity most directly support? A.Different environments require different adaptations. B.Humans have affected many different natural habitats. C.Rainfall and temperature will affect species diversity. D.Different habitats support different levels of diversity.

Option (D) is correct. The other responses would require additional data, which were not collected during the activity. Only the names and diversity levels of the habitats were available at the end of the activity.

In an outdoor class activity to investigate forces, students kicked soccer balls along thick grass, short grass, and smooth pavement. The teacher asked students to respond to the following statement. "Moving objects slow down and eventually stop because they have used up all the force from the initial push that started the motion." One student responded by saying, "I agree with this statement because the ball rolled after I kicked it. The force of my leg made the ball move. When that force was gone, the ball stopped rolling." Based on the student's response, the teacher should focus on which of the following concepts to correct the student's misunderstanding? A.A force can change the direction in which an object is moving. B.A push or pull can cause an object to start moving. C.A heavier object can require a larger force to start it moving. D.An object will continue moving at a constant velocity unless a force is exerted on it.

Option (D) is correct. The student may be confusing the concepts of energy and force. The student indicates understanding that the kick force affects the ball only when there is contact with the ball. But the student indicates lack of understanding of the concept that motion continues unchanged unless there is a force such as friction that slows down and stops motion.

A fifth-grade teacher is preparing for a lesson identifying similarities and differences among various racial, ethnic, and religious groups in the same country. The supporting material is a set of photographs of a variety of people labeled with the following attributes for each person: religion, music, clothing, and food preference. Which of the following activities is most likely to help the teacher achieve the lesson's objective? A.Making a collage out of photographs that represent one of the four attributes B.Sharing opinions about the photos during a small-group discussion C.Identifying photographs with a given attribute and doing further research on that attribute D.Sorting the photographs based on attributes and making comparisons among the people pictured

Option (D) is correct. The teacher should practice sorting the photographs and create questions for students that will prompt an understanding of the material. For example, people with the same color skin may have different religions and wear different clothing.

A fourth-grade teacher writes a list of professions in the community on a board and provides students with sticky notes to brainstorm attributes associated with each profession. Which of the following is the primary purpose of the lesson? A.Predetermining careers that are most suitable for the students B.Screening for career opportunities that the students are most familiar with C.Grooming the students to work for the best organizations D.Introducing students to the skills and qualities associated with different jobs

Option (D) is correct. This activity helps students understand and identify what skills and qualities are required of someone to be successful at different jobs.

Ms. Keane had her students plant seeds in pots of soil. As the seeds began to sprout, the students cared for them in class and observed their growth. The plants were placed in two locations, some directly on a sunny windowsill and some across the room on a table out of direct sunlight. After a weeklong break, the returning students found that some of their plants were wilted and some were dead. Ms. Keane asked the students to suggest a way to investigate why some of the plants had died and others had not. Which student plan will allow the students to collect data that can be used to determine why some of the plants died? Each plan includes collecting daily observations of the plants. A.Move all of the plants to the windowsill and water them the same amount each day. B.Move all of the plants to the dark and water half of them daily and half of them weekly. C.Examine the pots to determine whether the soil is dry and water each plant when the soil feels dry. D.Place new groups of plants in the original locations and water half of the plants in each group daily and water the other half weekly.

Option (D) is correct. This plan will allow students to test only the water variable while keeping the plants alive and allowing a comparison between groups of plants in different locations.

Groups of students in a second-grade class are each given a square sheet of aluminum foil and a basin of water. The groups are directed to manipulate the shape of the sheet of aluminum foil so that it floats in the basin of water, and to then manipulate the shape so that it sinks. Most of the groups begin making plans and testing various shapes, but one group is struggling to come up with a plan. Which of the following questions should the teacher ask to point the group toward a strategy to complete the assignment? A.How fast does aluminum foil absorb water? B.Why are we using aluminum foil rather than paper for this activity? C.Why are some boats made of wood while others are made of metal? D.Which is heavier, a hollow or a solid object of the same size and material?

Option (D) is correct. This question will most directly motivate students to shape the aluminum foil to enclose air by making a boat and to shape the foil to exclude air by crushing it into a solid object that sinks.

Students in two second-grade classrooms, both located on the east side of the school building, are learning about seeds and plants. The teachers in both classrooms have students place lima bean seeds into clear plastic bags and attach the bags to the classroom window so the students can watch the roots grow and the seeds germinate. Each day, the students record their observations and give their seeds a dropper of water. After two weeks, the lima beans in one classroom have all sprouted and grown into plants. The lima bean seeds in the other class have not all germinated and have not grown as quickly, and the teacher in that classroom asks the following question: "How would you explain why many of our seeds didn't grow as well as the seeds in the classroom next door did?" Select the student response that best demonstrates understanding of the concepts involved in the activity. A."The plants did not grow because the seeds did not germinate." B."The plants did not grow as well because we did not give them enough water." C."The plants did not grow because there was no soil and without the soil they cannot live." D."The plants did not grow as well because the tree outside our classroom blocks a lot of our sunlight."

Option (D) is correct. This response shows that the student understands the concept that the environment has an effect on the traits an organism develops and that the amount of sunlight was not the same in the two classrooms.

A first-grade teacher is introducing a unit on patriotism. The teacher makes ten stations around the room. Each station has a picture or artifact reflecting a patriotic symbol, custom, or celebration. Students are asked to visit each station and individually answer questions about each picture or artifact and turn in their answers to the teacher. Which of the following best identifies the goal of the station activity? A.Encouraging students to brainstorm project ideas on the new unit B.Guiding students' thought process to get them actively involved in the lesson C.Developing independent thinking and good work habits D.Identifying misconceptions to develop appropriate future lessons

Option (D) is correct. Through analyzing the student answers the teacher will be able to focus on correcting misconceptions and not waste time on information that students already know.

A grocery store sells both green grapes and red grapes for a regular price of $2.89 per pound. Nelson buys 1.5 pounds of green grapes and 2.25 pounds of red grapes at the store on a day when the regular price is reduced by $0.75 per pound. Which of the following expressions represents the amount, in dollars, that Nelson will pay for the grapes? A.1.5+2.25×2.89−0.75 B.(1.5+2.25)×2.89−0.75 C.1.5+2.25×(2.89−0.75) D.(1.5+2.25)×(2.89−0.75)

Option (D) is correct. To find the amount, in dollars, that Nelson will pay for the grapes, the total weight of the grapes, in pounds, needs to be multiplied by the reduced price of the grapes, in dollars. The total weight of the grapes, in pounds, is 1.5+2.25, and the reduced price of the grapes, in dollars, is 2.89−0.75, so the amount, in dollars, that Nelson will pay for the grapes is (1.5+2.25)×(2.89−0.75). The parentheses must be included in the expression as shown so that the total weight of the grapes will be multiplied by the reduced price of the grapes.

Ms. Pellegrino divides her class into four groups and gives each group a dozen bean seedlings to monitor for the semester. She asks each group to use the seedlings to investigate whether plants need sunlight to grow. Although she wants each group to develop its own procedure, she wants to prevent errors that will result in the collection of unusable data. Which of the following is a procedural error that Ms. Pellegrino should correct to prevent the collection of unusable data? A.Measuring plant height in inches instead of centimeters B.Not having a backup plan if all the students in the group are absent for one day C.Recording data on loose-leaf paper instead of in a bound laboratory notebook D.Measuring the final heights of the plants but not the initial heights

Option (D) is correct. To rely on changes in plant height as an indicator of plant growth, the students will need to measure both the initial height and final height of each plant in their investigation.

As part of a research project, a student discovered that during the 18th century many items were used as currency, including tobacco leaves. However, the tobacco leaves were soon replaced with paper money for transactions. Which of the following best represents why governments eventually began issuing paper money? A.Growing tobacco leaves became increasingly difficult B.Ensuring that the currency would not depreciate over time C.Using paper money was a novelty that attracted many traders D.Exchanging goods and services was easier with uniform currency

Option (D) is correct. Utilizing one form of paper money would be much easier to exchange than tobacco leaves and other forms of currency for the same goods and services.

Ms. Nero wants to introduce sentence frames that will help her students write stronger counterarguments in their persuasive essays. Which of the following sentence frames will be useful for the instructional purpose? Select all that apply. A.It might seem that _________. However, _____________. B.Some people think _________________. Despite this, ______________. C.In addition ________________________. Together with ____________.

Options (A) and (B) are correct. In the first sentences of (A) and (B), the phrases "It might seem that" and "Some people think" both signal a commonly held belief or an acknowledgement of the counterargument. In the second sentences of the options, "However" and "Despite this" indicate that the writer will refute the initial claim and present an opposing or different idea.

A fourth-grade geography teacher asks students to match different types of vegetation with one of the three regions in which they occur naturally: North Africa, South Asia, and the Arctic. Which TWO of the following geographic features of each region will students need to most closely consider to successfully complete the activity? A.The latitude of each region B.The human population of each region C.The average amount of rainfall in each region D.The tectonic plate on which each region exists E.The number of cities in each region

Options (A) and (C) are correct. If students can identify where regions are located in relation to the equator, they will be more likely to understand the climate of the region. Also, if students can identify the average amount of rainfall in a region, they will be more likely to understand the types of vegetation that the region can support.

In the partitive model of division, the quotient is the size of each group. In the measurement model of division, the quotient is the number of groups. Which of the following problems illustrates the measurement model of division? Select all that apply. A.Joe is making chocolate fudge and the recipe calls for 314 cups of sugar. Joe uses a 14-cup measuring cup to measure the sugar. How many times does Joe need to fill the measuring cup to measure the sugar needed for the recipe? B.314 cups of soup fills 14 of a container. How many cups of soup will it take to fill the whole container? C.A trail is 314 miles long and trail markers are placed at 14 -mile intervals along the trail. How many trail markers are placed along the trail?

Options (A) and (C) are correct. In the problem in option (A), the size of the measuring cup (the group) is given, and the problem asks for the number of times the measuring cup needs to be filled to measure the sugar. Similarly, in the problem in option (C), the size of the intervals (the group) is given, and the problem asks for the number of trail markers along the trail, which is determined by finding the number of 1/4-mile intervals along the trail. In contrast, the problem in option (B) asks for the number of cups of soup in the whole, or the one, container, so the problem is asking for the size of the whole group and is, therefore, using the partitive model of division.

Ms. Vargas asked her students to write an expression equivalent to 4(x−y). After substituting some values for x and y, a student named Andrew rewrote the expression as 4x−y. Andrew's expression is not equivalent to 4(x−y), but he thought his work was correct based on the substitutions he tried. For which of the following integer values of x and y would Andrew's expression appear to be correct? Select two choices. A.x=0 and y=0 B.x=0 and y≠0 C.x≠0 and y=0 D.x≠0 and x=y E.x≠0 and x=−y

Options (A) and (C) are correct. Since 4(x−y)=4x−4y, the expression 4(x−y) is equivalent to 4x−y only when 4y=y, and 4y=y only when y=0. The only options where y=0 are options (A) and (C).

Mr. Varela asked his students to define a square in terms of other two-dimensional geometric figures. Which two of the following student definitions precisely define a square? A.A square is a rectangle that has 4 sides of equal length. B.A square is a parallelogram that has 4 angles of equal measure. C.A square is a parallelogram that has 4 sides of equal length. D.A square is a rhombus that is also a rectangle. E.A square is a rectangle that is not a rhombus.

Options (A) and (D) are correct. A square is a quadrilateral with 4 sides of equal length and 4 angles of equal measure, whereas a rectangle is a quadrilateral with 4 angles of equal measure, a rhombus is a quadrilateral with 4 sides of equal length, and a parallelogram is a quadrilateral where opposite sides are parallel. Therefore, a rectangle that has 4 sides of equal length is a square, and a rhombus that is also a rectangle is a square, so options (A) and (D) are both precise definitions of a square. Option (B) describes a rectangle that is not necessarily a square, option (C) describes a rhombus that is not necessarily a square, and option (E) describes a rectangle that is not a square.

The kindergarten students are too short to use the wall-mounted water fountain in the hallway next to their classroom. Their teacher would like the class to formulate a solution to this problem. The teacher points out that answering questions can help solve problems and directs the class to come up with questions about the problem. Which two questions would most likely stimulate discussions that would lead to the class generating possible solutions to this problem? A.Can the water fountain be moved? B.Why are we thirsty all the time? C.Was this always the kindergarten classroom, or did our room use to have bigger kids? D.Is there enough space around the water fountain to place a step that people won't trip over? E.What's wrong with just taking a cup into the bathroom to get water from the sink?

Options (A) and (D) are correct. Answering (A) would determine whether a solution involving lowering the fountain is feasible. The answer to the question in (D) would determine whether adding a permanent or temporary step would solve the problem without creating additional problems.

Ms. Egbuniwe wants her students to analyze information and provide evidence that individuals inherit traits from their parents but with variation between the individuals and their parents. She would like to provide them with resources that contain appropriate evidence for an upper-elementary student. Of the following, which two would provide the best evidence? A.A picture of a female cocker spaniel, with her litter of eight puppies, some black, some golden, some brown B.A diagram showing the evolution of the modern horse over geologic time, including pictures of older, extinct species in the horse family C.A picture of a large tree, surrounded by shoots sprouting from its roots D.A diagram that shows a corn plant with red kernels and a corn plant with yellow kernels that have produced offspring containing both red and yellow kernels

Options (A) and (D) are correct. These two resources clearly show a parent, its offspring from sexual reproduction, and both similarities between and variation among all the individuals.

Ms. Stockton and her students are working on naming different polygons using their geometric attributes. She finds a suggestion online to use pattern blocks for this work. Which two of the following statements describe limitations of using pattern blocks for the purpose of naming polygons based on their attributes? A.The blocks could support the conclusion that all polygons are convex. B.The blocks do not clearly show how polygons can be composed of other polygons. C.The blocks could support the conclusion that all polygons can be composed of equilateral triangles. D.The blocks do not show which polygons have the defining attribute of having at least one set of parallel lines. E.The blocks could support the conclusion that an attribute of polygons is that they must have at least two sides of equal length.

Options (A) and (E) are correct. All the pattern blocks are convex polygons, and not all polygons are convex. Also, all the pattern blocks have at least two sides of equal length, and this is not an attribute of polygons. Option (B) is not correct because, for example, the yellow hexagon can be composed of two red trapezoids, and the blue rhombus can be composed of two green triangles. Option (C) is not correct because the tan rhombus and the orange square cannot be composed of equilateral triangles. Option (D) is not correct because the set of pattern blocks includes polygons that have the defining attribute of having at least one set of parallel lines.

Which three of the following words from the student's writing provide evidence that the student can identify each phoneme heard when the word is spoken? A."lik" for "like" B."udr" for "under" C."chre" for "tree" D."wn" for "win" E."priz" for "prize"

Options (A), (C), and (E) are correct. Option (A) is correct because the student heard the three phonemes in "like," which are /l/i/k/. Option (C) is correct because the response shows that the student heard the three phonemes in /t/r/E/. Although the student wrote "chr" instead of "tr," the sound made by these two blends is almost identical. Option (E) is correct because the student has heard the four phonemes in "prize," which are p/r/I/z.

Which three of the following word problems can be represented by a division equation that has an unknown quotient? A.Ms. Bronson works the same number of hours each day. After 8 days of work, she had worked 32 hours. How many hours does Ms. Bronson work each day? B.Mr. Kanagaki put tape around 6 windows before painting a room. He used 7 feet of tape for each window. How many feet of tape did he use? C.Micah used the same number of sheets of paper in each of 5 notebooks. He used 45 sheets of paper in all. How many sheets of paper did Micah use in each notebook? D.Each shelf in a school supply store has 8 packs of markers on it. Each pack has 12 markers in it. How many markers are on each shelf in the store? E.Trina gave each of 7 friends an equal number of beads to use to make a bracelet. She gave the friends a total of 63 beads. How many beads did she give to each friend?

Options (A), (C), and (E) are correct. The word problem in option (A) can be represented by the equation 32÷8=□32÷8=□, the word problem in option (C) can be represented by the equation 45÷5=□45÷5=□, and the word problem in option (E) can be represented by the equation 63÷7=□63÷7=□. The word problem in option (B) can be represented by the equation 7×6=□7×6=□ or the equation □÷6=7□÷6=7, and the word problem in option (D) can be represented by the equation 12×8=□12×8=□ or the equation □÷8=12□÷8=12; however, neither of these word problems can be represented by a division equation that has an unknown quotient.

Which three of the following words are phonetically regular? A.Beach B.Said C.Come D.Pine E.Stand

Options (A), (D), and (E) are correct. "Beach," "Pine," and "Stand" are phonetically regular because they have common phoneme-grapheme relationships and can be sounded out or decoded.

A second-grade teacher prepares for a beginning lesson focused on interpreting data from a map. Which TWO of the following map characteristics best support the teacher's lesson when used in a related activity? A.Shows a small area of land B.Contains a compass rose C.Provides a legend D.Includes a relevant title E.Represents sections of land in vibrant colors

Options (B) and (C) are correct. A compass rose critically assists students when interpreting a map to locate items presented in the legend. A legend contains symbols that represent objects on the map, which is critically important to helping students successfully interpret data on a map.

Mr. Russell is reading aloud a big book that has two lines of text per page. He is using a pointer to point to each word. Mr. Russell is reinforcing which of the following concepts of print as he reads aloud? Select all that apply. A.Concept of letter B.Directionality C.One-to-one matching

Options (B) and (C) are correct. Directionality (B), meaning how to read from left to right, is reinforced with Mr. Russell pointing to the text as he reads. One-to-one matching (C), showing the relationship between the word and the utterances that are spoken with the word, is also reinforced as Mr. Russell points to each individual word as it is read.

A grammatical error is present in which of the following sentences? Select all that apply. A.Do you know whose car this is? B.The cat lost it's toy mouse. C.Your the person I was talking about.

Options (B) and (C) are correct. In option (B), the usage of "it's," the contraction of "it is" is incorrect. This sentence should use "its," indicating possession, since the toy mouse belongs to the cat. In option (C), the usage of "your," a possessive pronoun, is incorrect; "you're," or the contraction of "you are," is the correct way to begin this sentence.

Mr. French's students are working on finding numbers less than 100 that are multiples of given one-digit numbers. When Mr. French asks them how they know when a number is a multiple of 6, one student, Crystal, says, "Even numbers are multiples of 6!" Mr. French wants to use two numbers to show Crystal that her description of multiples of 6 is incomplete and needs to be refined. Which of the following numbers are best for Mr. French to use for this purpose? Select two numbers. A.15 B.16 C.20 D.24 E.27 F.30

Options (B) and (C) are correct. The best numbers for Mr. French's purpose are even numbers that are not multiples of 6, and 16 and 20 are both even numbers, but they are not multiples of 6. Options (A) and (E) are incorrect because 15 and 27 are not even numbers, and options (D) and (F) are incorrect because 24 and 30 are both multiples of 6.

Mr. Johansen and his class are working on the mathematical objective shown. To assess what his students understand about the objective, Mr. Johansen plans to use base-ten blocks in a nonconventional way. He plans to first tell students which block will represent the unit and then ask students to determine what number is represented with the given base-ten blocks. Mr. Johansen wants to identify numbers that can be represented using only little cubes, rods, flats, and big cubes. If Mr. Johansen wants students to identify the number 32.6 from his representation, which two of the following base-ten blocks can he choose to represent the unit?

Options (B) and (C) are correct. The goal of the activity is to help students understand the relationship between place values depending on their relative position rather than relying on the intrinsic value of each base-ten block. If the rod is chosen to represent the unit, then the little cube represents 1/10 of the value of the rod (or 0.1 times the value of the rod), the flat represents 10 times the value of the rod, and the big cube represents 10 times the value of the flat (or 100 times the value of the rod). The number 32.6 can then be represented by using 3 flats, 2 rods, and 6 little cubes, so option (B) is correct. If the flat is chosen to represent the unit, then the rod represents 1/10 of the value of the flat, the little cube represents 1/10 of the value of the rod (or 1/100 of the value of the flat), and the big cube represents 10 times the value of the flat. The number 32.6 can then be represented by using 3 big cubes, 2 flats, and 6 rods, so option (C) is correct. If the little cube is chosen to represent the unit, then none of the base-ten blocks shown can be used to represent tenths, so option (A) is incorrect. Similarly, if the big cube is chosen to represent the unit, then none of the base-ten blocks shown can be used to represent tens, so option (D) is incorrect.

A kindergarten teacher provides students with a family tree diagram containing large leaves to represent various family members. The teachers asks students to draw a picture of a family member in each leaf. The students then present their trees to their classmates. Which TWO of the following best identify the purpose of the activity? A.Learning to play and share cooperatively B.Understanding family structures C.Understanding the roles of individuals in the community D.Learning to identify their family's ethnic backgrounds E.Understanding everyone is part of a family

Options (B) and (E) are correct. Drawing pictures into a family tree and placing each family member on the tree help to give students an idea of where each family member fits into the overall family structure, and by presenting the family trees to the class the students will see that all students are part of a family, even though the family structures may be different.

Which three of the following words contain a digraph? A.True B.Ship C.Bath D.Bread E.Fish

Options (B), (C), and (E) are correct. They each contain digraphs, in which two or more consonants are combined to make one sound. In (B) and (E) the digraph is the "sh" in "ship" and "fish." In (C) the digraph is the "th" in "bath."

Mr. Benner places a row of 5 cubes on a student's desk and asks the student, Chanel, how many cubes are on the desk. As Chanel points at the cubes one by one from left to right, she counts, saying, "One, two, three, four, five." Then she says, "There are five cubes!" Mr. Benner then asks Chanel to pick up the third cube in the row. As Chanel points at three cubes one by one from left to right, she counts, saying, "One, two, three." She stops, then picks up the three cubes, and gives them to Mr. Benner. Chanel has demonstrated evidence of understanding which two of the following mathematical ideas or skills? A.Using numerals to describe quantities B.Recognizing a small quantity by sight C.Counting out a particular quantity from a larger set D.Understanding that the last word count indicates the amount of objects in the set E.Understanding that ordinal numbers refer to the position of an object in an ordered set

Options (C) and (D) are correct. Chanel first counts the cubes one by one and then she states that there are 5 cubes. Her work demonstrates that she understands that when counting a set of objects, the last word count indicates the cardinality of the set (that is, the number of objects in the set), so option (D) is correct. When she is asked to pick the third cube in the row, she is able to count and stop at three, thus demonstrating that she can count out a quantity from a larger set, so option (C) is correct. However, since she does not pick up the third cube counted, but instead picks up the three cubes she counted, she shows that she does not yet understand that the ordinal number "third" refers to the position of third counted cube, which means that option (E) is incorrect. Also, Chanel does not demonstrate the ability to recognize a small quantity by sight since she counts one by one, so option (B) is incorrect. Finally, she is not asked to record the numeral that describes the number of cubes on her desk, so there is no evidence that Chanel can use numerals to describe quantities, which means that option (A) is incorrect.

Mr. Wilson's upper elementary class is researching the positive and negative effects of pesticide and herbicide use in local agriculture. The class has already spent many hours searching on the Internet but has collected very little information. He would like to provide two additional opportunities for the students to collect information that would be the most useful. Which two activities would provide the best opportunities to collect useful information? A.Mr. Wilson can schedule a trip to a local soybean farm. The students will tour the farm and interview the farmer for information on his farming practices. B.Mr. Wilson can schedule a trip to a pesticide production plant. The students will have a chance to interview the scientists employed by the company and receive materials that may not be present on the Internet. C.Mr. Wilson can meet with the school librarian, outlining his objectives and requirements. He can then bring his class to the library for several research periods after the librarian has had a chance to collect and lay out relevant journals, newspapers, and books. D.Mr. Wilson can schedule a panel discussion with representatives from the Environmental Protection Agency, U.S. Department of Agriculture, and the local extension service. They can provide materials that may not be available on the Internet, provide suggestions for additional resources, and answer student questions.

Options (C) and (D) are correct. They describe good sources of unbiased, authoritative information. The other options would present limited, possibly biased, information.

Which TWO of the following history lesson activities would best facilitate the use of chronological thinking skills? A.Determining the publication dates of historical texts in the school library B.Examining multiple eyewitness perspectives of a historical event C.Sequencing a series of events leading up to the American Revolution on a time line D.Describing the foreign policy objectives of current world leaders E.Identifying events that contributed to trends in a graph of twentieth-century United States immigration data

Options (C) and (E) are correct. Chronological thinking skills include the ability to place events in chronological sequence, such as on a time line, and the ability to identify factors that contributed to changes across long periods of time, such as in data graphs.

Which two of the following words can best be decoded by looking for a root word within the larger word? A.Snitch B.Colossal C.Endanger D.Fashion E.Lovely

Options (C) and (E) are correct. The larger word "endanger" can be decoded by finding the root word "danger," and the larger word "lovely" can be decoded by identifying the root word "love."

Which two of the following inequalities are true? A.0.56>0.605 B.0.065>0.56 C.0.56>0.506 D.0.605<0.056 E.0.506<0.65 F.0.65<0.605

Options (C) and (E) are correct. To compare these decimal numbers, first compare the digits in the tenths place—the decimal number with the greater digit in the tenths place will be the greater number. If the digits in the tenths place are the same, compare the digits in the hundredths place to determine which decimal number is greater. This process can be continued as needed. Another way to compare these decimal numbers is to write each number to the thousandths place to make the comparison easier. For example, in option (C), 0.560>0.506, so 0.56>0.506.

Stages of Spelling Development

Precommunicative stage The child uses symbols from the alphabet but shows no knowledge of letter-sound correspondences. The child may also lack knowledge of the entire alphabet, the distinction between upper- and lower-case letters, and the left-to-right direction of English orthography. Semiphonetic stage The child begins to understand letter-sound correspondence ? that sounds are assigned to letters. At this stage, the child often employs rudimentary logic, using single letters, for example, to represent words, sounds, and syllables (e.g., U for you). Phonetic stage The child uses a letter or group of letters to represent every speech sound that they hear in a word. Although some of their choices do not conform to conventional English spelling, they are systematic and easily understood. Examples are KOM for come and EN for in. Transitional stage The speller begins to assimilate the conventional alternative for representing sounds, moving from a dependence on phonology (sound) for representing words to a reliance on visual representation and an understanding of the structure of words. Some examples are EGUL for eagle and HIGHEKED for hiked. Correct stage The speller knows the English orthographic system and its basic rules. The correct speller fundamentally understands how to deal with such things as prefixes and suffixes, silent consonants, alternative spellings, and irregular spellings. A large number of learned words are accumulated, and the speller recognizes incorrect forms. The child's generalizations about spelling and knowledge of exceptions are usually correct.

Anthony is presenting evidence that the matter taken in by plants for growth and repair is mostly carbon dioxide (a gas) and water. He has recently completed a series of inquiries and is using his results to support his claim. "My investigations involved growing plants under different levels of light and watering. All the plants started out weighing 25 grams. At the end of the experiment, the plants that received the most light grew the most, ending up at about 110 grams, and the ones with the least amount of water and light grew the least, ending at about 50 grams. The pots all started out with 200 grams of soil, and each pot lost about 5 to 10 grams of soil." Anthony has made the list of five reasons to support his claim that plants use light energy to make matter from air and water. Select the three reasons that would be considered valid evidence. "Based on this information, I think that plants make materials from air and water, for the following reasons: A.The plants given more water grew more. B.The fact that more light made a difference means that the plant is using photosynthesis. C.My friend told me that photosynthesis uses carbon dioxide, so that must be the gas that was used. D.After watering there were small particles of soil under the pots, which is why the soil in the pot weighed less at the end. E.Unfortunately, since gas has no weight, there is no way to test this."

Reasons A, B, and D are correct and would be considered valid evidence.

Semantic Cues

Refer to the meaning in language that assists in comprehending texts, including words, speech, signs, symbols, and other meaning-bearing forms. Semantic cues involve the learners' prior knowledge of language, text, and visual media, and their prior life experiences.

One-to-one matching

Showing the relationship between the word and the utterances that are spoken with the word.

Identify the number of phonemes the following words contain. Knock Strict

The answers (3) and (6) are correct. Knock has 3 phonemes: /n/o/k/. Strict has 6 phonemes: /s/t/r/i/k/t/.

Phoneme to Grapheme Relationships

The connections between the sounds in words and the letters that are used to represent those sounds.

A chef at a restaurant uses 1/5 liter of lemon juice and 3/10 liter of teriyaki sauce to make a marinade for 2 kilograms of salmon. How many liters of marinade does the chef use per kilogram of salmon? Give your answer as a fraction.

The correct answer is 14. The chef uses 1/5+3/10=2/10+3/10=5/10=1/2 liter of marinade for 2 kilograms of salmon. This means that the chef uses 1/2÷2=1/4 liter of marinade per kilogram of salmon.

Joshua walks the length of each of three trails on a hike. The first trail is 3.6 kilometers long. The second trail is 3.7 kilometers long. The third trail is 600 meters shorter than the sum of the lengths of the first two trails. Joshua walks at an average speed of 3 kilometers per hour over the course of the entire hike. How many minutes does it take Joshua to complete his hike?

The correct answer is 280 minutes. Since 600 meters is equivalent to 0.6 kilometers, the third trail is 3.6+3.7−0.6=6.7 kilometers long. Therefore, Joshua walked a total distance of 3.6+3.7+6.7=14 kilometers on his hike. Since Joshua walks at an average speed of 3 kilometers per hour and there are 60 minutes in an hour, the proportion 3 kilometers/60 minutes=14 kilometers/x minutes can be used to find how many minutes it takes Joshua to complete his hike. Based on the proportion, 3x=(14)(60), and since (14)(60)=840, x=840/3=280, which means that it takes Joshua 280 minutes to complete his hike.

Prosody

The rhythm, stress, and intonation of speech.

Phoneme

The smallest unit of sound in a word. /k/

Rime

The string of letters that follow the onset which contains the vowel and any final consonants.

Grapheme

The way we write a phoneme/ /k/ can be written 5 ways, c k ck qu ch

Tiers of Vocabulary

Tier 1: everyday words (basic, commonly used) Tier 2: words used in several different subjects (general academic) Tier 3: Domain specific words (discipline-specific)

Consonant Digraphs

Two or more consonants that, together represent one sound. For example, the consonants "p" and "h" form the grapheme ph that can represent the /f/ sound.

Consonant blends

When two or more consonants are blended together, but each sound may be heard in the blend. (bl, br, cl, cr, dr, fr, tr, fl, gl, gr, pl, pr, sl, sm, sp, st)


Related study sets

ch. 1 - mental health and mental illness

View Set

Audit Ch 15, CH 15 (TB), AC 432 Chp 15, chapter 17, Audit Ch 15, Audit Ch 15, Chapter 15, Audit Ch 15, Audit Ch 15, Chapter 15 Exam 2, Audit Ch 15, Audit Ch. 8 MC, Auditing Profession and Planning

View Set

Macroeconomics ch. 12 (pt 1&2 not on ashtyns and 3-5)

View Set

D076 - Finance by ME (Lesson Checks)

View Set

The Practice of Mental Health/Psychiatric Nursing

View Set

Exam #2 Knowledge Check Questions

View Set

Biochemistry Exam 2: Chapter 9 (Nucleic Acids: How Structure Conveys Information)

View Set